Nat-Res Cases 1

You might also like

Download as pdf or txt
Download as pdf or txt
You are on page 1of 43

NAT RES & ENVI LAW CASES 2A_2023

[G.R. No. 48321. August 31, 1946. ]

OH CHO, applicant-appellee, v. THE DIRECTOR OF LANDS, Oppositor-Appellant. PADILLA, J.:

Solicitor General Roman Ozaeta and Assistant Solicitor General Rafael Amparo,
for Appellant. This is an appeal from a judgment decreeing the registration of a residential lot located in
the municipality of Guinayangan, Province of Tayabas, in the name of the applicant.
Vicente Constantino, for Appellee.
The opposition of the Director of Lands is based on the applicant’s lack of title to the lot,
Ferrier, Gomez & Sotelo and J.T. Chuidian as amici curiae. and on his disqualification, as alien, from acquiring lands of the public domain.

SYLLABUS The applicant, who is an alien, and his predecessors in interest have been in open,
continuous, exclusive and notorious possession of the lot from 1880 to the filing of the
1. LAND REGISTRATION; PUBLIC LANDS; WHAT LANDS BELONG TO PUBLIC application for registration on January 17, 1940.
DOMAIN; EXCEPTION. — All lands that were not acquired from the Government, either
by purchase or by grant, belong to the public domain. An exception to the rule would be The Solicitor General reiterates the second objection of the opponent and adds that the
any land that should have been in the possession of an occupant and of his predecessors lower court committed an error in not declaring null and void the sale of the lot to the
in interest since time immemorial, for such possession would justify the presumption that applicant.
the land had never been part of the public domain or that it had been a private property
even before the Spanish conquest. The applicant invokes the Land Registration Act (Act No. 496), or should it not be
applicable to the case, then he would apply for the benefits of the Public Land Act (C.A.
2. ID., ID.; ID.; ID.; CASE AT BAR. — The earliest possession of the lot by the first No. 141).
predecessor in interest of the applicant for registration began in 1880. Held: He does not
come under the exception. The applicant failed to show that he has title to the lot that may be confirmed under the
Land Registration Act. He failed to show that he or any of his predecessors in interest had
3. ID.; PUBLIC LAND ACT, REGISTRATION UNDER; ALIEN DISQUALIFIED. — An alien acquired the lot from the Government, either by purchase or by grant, under the laws,
is not entitled to a decree of registration under the provisions of the Public Land Act, orders and decrees promulgated by the Spanish Government in the Philippines, or by
because he is disqualified from acquiring lands of the public domain. possessory information under the Mortgage Law (section 19, Act 496). All lands that were
not acquired from the Government, either by purchase or by grant, belong to the public
4. ID.; PUBLIC LAND ACT, BENEFITS OF; CONDITION PRECEDENT; CASE AT BAR. domain. An exception to the rule would be any land that should have been in the
— The benefits provided in the Public Land Act for applicant’s immediate predecessors in possession of an occupant and of his predecessors in interest since time immemorial, for
interest are or constitute a grant or concession by the State; and before they could acquire such possession would justify the presumption that the land had never been part of the
any right under such benefits, the applicant’s immediate predecessors in interest should public domain or that it had been a private property even before the Spanish conquest.
comply with the condition precedent for the grant of such benefits. The condition precedent (Carino v. Insular Government, 212 U.S., 449; 53 Law. ed., 594.) The applicant does not
is to apply for the registration of the land of which they had been in possession at least come under the exception, for the earliest possession of the lot by his first predecessor in
since July 26, 1894. This the applicant’s immediate predecessors in interest failed to do. interest began in 1880.
They did not have any vested right in the lot amounting to title which was transmissible to
the applicant. The only right,if it may thus be called, is their possession of the lot which, As the applicant failed to show title to the lot, the next question is whether he is entitled to
tacked to that of their predecessors in interest, may be availed of by a qualified person to a decree of registration thereof under the provisions of the Public Land Act (C. A. No. 141).
apply for its registration but not by a person as the applicant who is disqualified. Under the provisions of the Act invoked by the applicant, he is not entitled to a decree of
registration of the lot, because he is an alien disqualified from acquiring lands of the public
domain (sections 48, 49, C. A. No. 141).
DECISION

1|Page
NAT RES & ENVI LAW CASES 2A_2023
As the applicant failed to prove title to the lot and has invoked the provisions of the Public land in question is susceptible of cultivation and may be converted into an orchard or
Land Act, it seems unnecessary to make pronouncement in this case on the nature, garden. Rodolfo Tiquia, inspector of the Bureau of Lands, testifying as a witness for the
character or classification of the lot sought to be registered. government, stated that the land, notwithstanding the use to which it is actually devoted,
is agricultural land in accordance with an opinion rendered in 1939 by the Secretary of
It may be argued that under the provisions of the Public Land Act the applicant’s immediate Justice. The pertinent part of said opinion, penned by Secretary Jose Abad Santos, later
predecessors in interest would have been entitled to a decree of registration of the lot had Chief Justice of the Supreme Court, is as follows:j
they applied for its registration; and that he having purchased or acquired it, the right of his
immediate predecessors in interest to a decree of registration must be deemed also to "1. Whether or not the phrase ’public agricultural land’ in section 1, Article XII, of
have been acquired by him. The benefits provided in the Public Land Act for applicant’s the Constitution may be interpreted to include residential, commercial or industrial
immediate predecessors in interest are or constitute a grant or concession by the State; lots for purposes of their disposition.
and before they could acquire any right under such benefits, the applicant’s immediate
predecessors in interest should comply with the condition precedent for the grant of such "1. Section 1, Article XII of the Constitution classifies lands of the public domain in the
benefits. The condition precedent is to apply for the registration of the land of which they Philippines into agricultural, timber and mineral. This is the basic classification adopted
had been in possession at least since July 26, 1894. This the applicant’s immediate since the enactment of the Act of Congress of July 1, 1902, known as the Philippine Bill.
predecessors in interest failed to do. They did not have any vested right in the lot At the time of the adoption of the Constitution of the Philippines, the term ’agricultural public
amounting to title which was transmissible to the applicant. The only right, if it may thus be lands’ had, therefore, acquired a technical meaning in our public laws. The Supreme Court
called, is their possession of the lot which, tacked to that of their predecessors in interest, of the Philippines in the leading case of Mapa v. Insular Government, 10 Phil., 175, held
may be availed of by a qualified person to apply for its registration but not by a person as that the phrase ’agricultural public lands’ means those public lands acquired from Spain
the applicant who is disqualified. which are neither timber nor mineral lands. This definition has been followed by our
Supreme Court in many subsequent cases. (Montano v. Ins. Gov’t., 12 Phil., 572, 574;
It is urged that the sale of the lot to the applicant should have been declared null and void. Santiago v. Ins. Gov’t., 12 Phil., 593; Ibañes de Aldecoa v. Ins. Gov’t., 13 Phil., 159; Ins.
In a suit between vendor and vendee for the annulment of the sale, such pronouncement Gov’t. v. Aldecoa & Co., 19 Phil., 505, 516; Mercado v. Collector of Internal Revenue, 32
would be necessary, if the court were of the opinion that it is void. It is not necessary in Phil., 271, 276; Molina v. Rafferty, 38 Phil., 167, 170; Ramos v. Director of Lands, 39 Phil.,
this case where the vendors do not even object to the application filed by the vendee. 175, 181; Jocson v. Director of Forestry, 39 Phil., 560, 564; and Ankron v. Government of
the Philippines, 40 Phil., 10, 14.)
Accordingly, judgment is reversed and the application for registration dismissed,
without costs. "Residential, commercial or industrial lots forming part of the public domain must have to
be included in one or more of these classes. Clearly, they are neither timber nor mineral,
Moran, C.J., Feria, Pablo, Hilado and Bengzon, JJ., concur. of necessity, therefore, they must be classified as agricultural.

Separate Opinions "Viewed from another angle, it has been held that in determining whether lands are
agricultural or not, the character of the lands is the test (Odell v. Durant, 62 N. W., 524;
Lerch v. Missoula Brick & Tile Co., 123 p., 25). In other words, it is the susceptibility of the
PERFECTO, J., concurring:chanrob1es virtual 1aw library land to cultivation for agricultural purposes by ordinary farming methods which determines
whether it is agricultural or not (State v. Stewart, 190, p., 129)."cralaw virtua1aw library
Oh Cho, a citizen of the Republic of China, purchased in 1938 from Antonio, Luis and
Rafael Lagdameo a parcel of land located in the residential district of Guinayangan, Judge Pedro Magsalin, of the Court of First Instance of Tayabas, rendered a decision on
Tayabas, which has been in the continuous, public, and adverse possession of their August 15, 1940, overruling the opposition without much explanation and decreeing the
predecessors in interest as far back as 1880. On June 17, 1940, Oh Cho applied for the registration prayed for by the applicant. The Director of Lands appealed from the decision,
registration of said parcel of land. The Director of Lands opposed the application because, and the Solicitor General appearing for appellant, maintains that the applicant, not being
among other grounds, the Constitution prohibits aliens from acquiring public or private a citizen of the Philippines, is disqualified to buy or acquire the parcel of land in question
agricultural lands. and that the purchase made in 1938 is null and void.

One of the witnesses for the applicant, on cross-examination, expressly admitted that the This is the question squarely submitted to us for decision. The majority, although reversing
2|Page
NAT RES & ENVI LAW CASES 2A_2023
the lower court’s decision and dismissing the application with which we agree, abstained first public land law of the Philippines. As therein used, the phrase was expressly given by
from declaring null and void the purchase made by Oh Cho in 1938 as prayed for by the the Philippine Commission the same meaning intended for it by Congress as interpreted
appellant. We deem it necessary to state our opinion on the important question raised by in the case of Mapa v. Insular Government, supra. This is self-evident from a reading of
the Solicitor General. Having been squarely raised, it must be squarely decided. sections 1, 10, 32, and 64 (subsection 6 of Act No. 926). Whenever the phrase ’agricultural
public lands’ is used in any of said sections, it is invariably followed by the qualifications ’as
The Solicitor General argued in his brief as follows:jgc:chanrobles.com.ph defined by said Act of Congress of July first, nineteen hundred and two.’

"1. The lower court erred in decreeing the registration of the lot in question in favor of the "More specifically, in the case of Ibañez de Aldecoa v. Insular Government, supra, the
applicant who, according to his own voluntary admission, is a citizen of the Chinese Supreme Court held that a residential or building lot, forming part of the public domain, is
Republic. agricultural land, irrespective of the fact that it is not actually used for purposes of
agriculture for the simple reason that it is susceptible of cultivation and may be converted
"(a) The phrase ’agricultural land’ as used in the Act of Congress of July 1, 1902, and in into a rural estate, and because when a land is not mineral or forestal in its nature it must
the Public Land Act includes residential lots. necessarily be included within the classification of agricultural land. Because of the special
applicability of the doctrine laid down in said case, we quote at some length from the
"In this jurisdiction lands of the public domain suitable for residential purposes are decision therein rendered:jgc:chanrobles.com.ph
considered agricultural lands under the Public Land Law. The phrase ’agricultural public
lands’ has a well settled judicial definition. It was used for the first time in the Act of "‘The question set up in these proceedings by virtue of the appeal interposed by counsel
Congress of July 1, 1902, known as the Philippine Bill. It means those public lands for Juan Ibañez de Aldecoa, is whether or not a parcel of land that is susceptible of being
acquired from Spain which are neither mineral nor timber lands (Mapa v. Insular cultivated, and ceasing to be agricultural land, was converted into a building lot, is subject
Government, 10 Phil., 175; Montano v. Insular Government, 12 Phil., 572; Ibañez de to the legal provisions in force regarding Government public lands which may be alienated
Aldecoa v. Insular Government, 13 Phil., 159; Ramos v. Director of Lands, 39 Phil, 175; in favor of private individuals or corporations. . . .
Jocson v. Director of Forestry, 39 Phil., 560; Ankron v. Government of the Philippine
Islands, 40 Phil., 10). In the case of Mapa v. Insular Government, supra, the Supreme x x x
Court, in defining the meaning and scope of that pharase from the context of sections 13
and 15 of that Act, said:
"‘Hence, any parcel of land or building lot is susceptible of cultivation, and may be
"The phrase ’agricultural public lands’ as defined by the Act of Congress of July 1, 1902, converted into a field, and planted with all kinds of vegetation; for this reason, where land
which phrase is also to be found in several sections of the Public Land Act (No. 926) means is not mining or forestal in its nature, it must necessarily be included within the classification
those public lands acquired from Spain which are neither mineral nor timber lands. of agricultural land, not because it is actually used for the purposes of agriculture, but
because it was originally agricultural and may again become so under other circumstances:
x x x besides the Act of Congress (of July 1, 1902)) contains only three classifications, and
makes no special provision with respect to building lots or urban land that have ceased to
be agricultural land. . . .
"‘We hold that there is to be found in the act of Congress a definition of the phrase
"agricultural public lands," and after a careful consideration of the question we are satisfied x x x
that the only definition which exists in said Act is the definition adopted by the court below
Section 13 says that the Government shall "make rules and regulations for the lease, sale,
or other dispositions of public lands other than timber or mineral lands." To our minds that "‘From the language of the foregoing provisions of the law, it is deduced that, with the
is the only definition that can be said to be given to agricultural lands. In other words, that exception of those comprised within the mineral and timber zone, all lands owned by the
the phrase "agricultural land" as used in Act No. 926 means those public lands acquired State or by the sovereign nation are public in character, and per se alienable and, provided
from Spain which are not timber or mineral lands. . . .’ (Mapa v. Insular Government, 10 they are not destined to the use of public in general or reserved by the Government in
Phil., 175, 178, 182, emphasis added.) accordance with law, they may be acquired by any private or juridical person; and
considering their origin and primitive state and the general uses to which they are accorded,
This phrase ’agricultural public lands’ was subsequently used in Act No. 926, which is the they are called agricultural lands, urban lands and building lots being included in this
3|Page
NAT RES & ENVI LAW CASES 2A_2023
classification for the purpose of distinguishing rural and urban estates from mineral and because only agricultural public lands are subject to alienation or disposition under section
timber lands; the transformation they may have undergone is no obstacle to such 1, Article XII of the Constitution. A contrary view would necessarily create a conflict
classification as the possessors thereof may again convert them into rural estates.’ (Ibañez between Commonwealth Act No. 141 and section 1 of Article XII of the Constitution, and
de Aldecoa v. Insular Government 13 Phil., 161, 163, 164, 165, 166; emphasis added.) such conflict should be avoided, if possible, and said Act construed in the light of the
fundamental provisions of the Constitution and in entire harmony therewith.
"(b) Under the Constitution and Commonwealth Act No. 141 (Public Land Act), the
phrase ’public agricultural land’ includes lands of the public domain suitable for residential "‘Another universal principle applied in considering constitutional questions is, that an Act
purposes. will be so construed, if possible, as to avoid conflict with the Constitution, although such a
construction may not be the most obvious or natural one. "The courts may resort to an
"Section 1. Article XII of the Constitution, reads as follows:jgc:chanrobles.com.ph implication to sustain a statute, but not to destroy it." But the courts cannot go beyond the
province of legitimate construction, in order to save a statute; and where the meaning is
"‘All agricultural timber, and mineral lands of the public domain, waters, minerals, coal, plain, words cannot be read into it or out of it for that purpose.’ (1 Sutherland, Statutory
petroleum, and other mineral oils, all forces of potential enerby, and other natural Construction, pp. 135, 136.)
resources of the Philippines belong to the State, and their disposition, exploitation,
development, or utilization shall be limited to citizens of the Philippines, or to corporations "In view of the fact that more than one year after the adoption of the Constitution the
or associations at least sixty per centum of the capital of which is owned by such citizens, National Assembly revised the Public Land Law and passed Commonwealth Act No. 141,
subject to any existing right, grant, lease, or concession at the time of the inauguration of which is a compilation of the laws relative to lands of the public domain and the
the Government established under this Constitution. Natural resources, with the exception amendments thereto, the statute so revised and compiled must necessarily conform to the
of public agricultural land, shall not be alienated . . .’ (Emphasis added.) . Constitution.

"Under the above-quoted provision, the disposition, exploitation, development or utilization "‘Where the legislature has revised a statute after a Constitution has been adopted, such
of the natural resources, including agricultural lands of the public domain, is limited to a revision is to be regarded as a legislative construction that the statute so revised
citizens of the Philippines or to the corporations or associations therein mentioned. It also conforms to the Constitution.’ (59 C. J., 1102; emphasis added.)
clearly appears from said provision that natural resources, with the exception of public
agricultural land, are not subject to alienation. "By way of illustration, let us suppose that a piece or tract of public land has been classifed
pursuant to section 9 of Commonwealth Act No. 141 as residential land. If, by reason of
"On November 7, 1936, or more than one year after the adoption of the Constitution, this classification, it is maintained that said land has ceased to be agricultural public land,
Commonwealth Act No. 141, known as the Public Land Act, was approved. Under this Act it will no longer be subject to alienation or disposition by reason of the constitutional
the lands of the public domain have been classified into three divisions: (a) alienable or provision that only agricultural lands are alienable; and yet such residential lot is alienable
disposable, (b) timber, and (c) mineral lands. The lands designated alienable or disposable under sections 58, 59, and 60 of Commonwealth Act No. 141 to citizens of the Philippines
correspond to the lands designated in the Constitution as public agricultural lands, or to corporations or associations mentioned in section 1, Article XII of the Constitution.
because under section 1, Article XII, public agricultural lands are the only natural resources Therefore, the classification of public agricultural lands into various subdivisions is only for
of the country which are subject to alienation or disposition. purposes of administration, alienation or disposition, but it does not destroy the inherent
nature of all such lands as public agricultural lands.
"Section 9 of Commonwealth Act No. 141 provides that the alienable or disposable public
lands shall be classified, according to the use or purposes to which they are destined, into "(c) Judicial interpretation of doubtful clause or phrase used in the law, controlling.
agricultural, residential, commercial, industrial, etc., lands. At first blush it would seem that
under this classification residential land is different from agricultural land. The difference, "The judicial interpretation given to the phrase ’public agricultural land’ is a sufficient
however, is more apparent than real.’Public agricultural lands’ as that phrase is used in authority for giving the same interpretation to that phrase as used in subsequent legislation,
the Constitution means alienable lands of the public domain and therefore this phrase is and this is especially so in view of the length of time during which this interpretation has
equivalent to the lands classified by Commonwealth Act No. 141 as alienable or disposable. been maintained by the courts. On this point Sutherland has the following to
The classification provided in section 9 is only for purposes of administration and say:jgc:chanrobles.com.ph
disposition, according to the purposes to which said lands are especially adapted. But
notwithstanding this classification all of said lands are essentially agricultural public lands "‘When a judicial interpretation has once been put upon a clause, expressed in a vague
4|Page
NAT RES & ENVI LAW CASES 2A_2023
manner by the legislature, and difficult to be understood, that ought of itself to be a "‘Legislative adoption of judicial construction. — In the adoption of the code, the legislature
sufficient authority for adopting the same construction. Buller, J., said: "We find one is presumed to have known the judicial construction which had been placed on the former
solemn determination of these doubtful expressions in the statute, and as that construction statutes; and therefore the reenactment in the code or general revision of provisions
has since prevailed, there is no reason why we should now put another construction on substantially the same as those contained in the former statutes is a legislative adoption
the act on account of any supposed change of convenience." This rule of construction will of their known judicial construction, unless a contrary intent is clearly manifest. So the fact
hold good even if the court be of opinion that the practical construction is erroneous; so that the revisers eliminated statutory language after it had been judicially construed shows
that if the matter were res integra the court would adopt a different construction. Lord that they had such construction in view.’ (59 C. J., 1102.) .
Cairns said: "I think that with regard to statutes . . . it is desirable not so much that the
principle of the decision should be capable at all times of justification, as that the law should "II. The lower court erred in not declaring null and void the sale of land to the appellant
be settled, and should, when once settled, be maintained without any danger of vacillation (appellee).
or uncertainty." Judicial usage and practive will have weight, and when continued for a
long time will be sustained through carried beyond the fair support of the statute." (II Lewis’ "Granting that the land in question has ceased to be a part of the lands of the public domain
Sutherland Statutory Construction, pp. 892, 893.) by reason of the long, continuous, public and adverse possession of the applicant’s
predecessors in interest, and that the latter had performed all the conditions essential to a
"‘An important consideration affecting the weight of contemporary judicial construction is Government grant and were entitled to a certificate of title under section 48, subsection
the length of time it has continued. It is adopted, and derives great force from being (b), of Commonwealth Act No. 141, still the sale of said land on December 8, 1938, to the
adopted, soon after the enactment of the law. It may be, and is presumed, that the applicant as evidenced by Exhibits B and C, was null and void for being contrary to section
legislative sense of its policy, and of its true scope and meaning, permeates the judiciary 5, Article XII of the Constitution, which reads as follows:jgc:chanrobles.com.ph
and controls its exposition. Having received at that time a construction which is for the time
settled, accepted, and thereafter followed or acted upon, it has the sanction of the authority "Save in cases of hereditary succession, no private agricultural land shall be transferred
appointed to expound the law, just and correct conclusions; when reached, they are, or assigned except to individuals, corporations, or associations qualified to acquire or hold
moreover, within the strongest reasons on which is founded the maxim of stare decisis. lands of the public domain in the Philippines.’
Such a construction is publicly given, and the subsequent silence of the legislature is
strong evidence of acquiescence, though not conclusive. . . . (II Lewis’ Sutherland "The applicant, being a Chinese citizen, is disqualified to acquire or hold lands of the public
Statutory Construction, pp. 894, 895.) domain (section 1, Article XII of the Constitution; sections 12, 22, 23, 33, 44, 48,
Commonwealth Act No. 141), and consequently also disqualified to buy and acquire
"Furthermore, when the phrase ’public agricultural land’ was used in section 1 of Article private agriculture land.
XII of the Constitution, it is presumed that it was so used with the same judicial meaning
therefor given to it and therefore the meaning of the phrase, as used in the Constitution, "In view of the well settled judicial meaning of the phrase ’public agricultural land,’ as
includes residential lands and other lands of the public domain, but excludes mineral and hereinbefore demonstrated, the phrase ’private agricultural land,’ as used in the above
timber lands. quoted provision, can onlymean land of private ownership, whether agricultural, residential,
commercial or industrial. And this is necessarily so, because the phrase ’agricultural land’
"‘Adoption of provisions previously construed — ad. Previous construction by Courts. — used in the Constitution and in the Public Land Law must be given the same uniform
Where a statute that has been construed by the courts of last resort has been reenacted meaning, to wit, any land of the public domain or any land of private ownership, which is
in same, or substantially the same, terms, the legislature is presumed to have been familiar neither mineral nor forestal.
with its construction, and to have adopted it as a part of the law, unless a contrary intent
clearly appears, or a different construction is expressly provided for; and the same rule "‘A word or phrase repeated in a statute wil bear the same meaning throughtout the statute,
applies in the construction of a statute enacted after a similar or cognate statute has been unless a different intention appears. . . .Where words have been long used in a technical
judicially construed. So where words or phrases employed in a new statute have been sense and have been judicially construed to have a certain meaning, and have been
construed by the courts to have been used in a particular sense in a previous statute on adopted by the legislature as having a certain meaning prior to a particular statute in which
the same subject, or one analogous to it, they are presumed, in the absence of clearly they are used, the rule of construction requires that the words used in such statute should
expressed intent to the contrary, to be used in the same sense in the new statute as in the be construed according to the sense in which they have been so previously used, although
previous statute.’ (59 C. J., 1061-1063.) to the sense in which they have been so previously used, although that sense may vary
from the strict literal meaning of the words.’ (II Sutherland, Statutory Construction, p. 758)
5|Page
NAT RES & ENVI LAW CASES 2A_2023

"This interpretation is in harmony with the nationalistic policy, spirit and purpose of our From the same book of Delegate Aruego, we quote:jgc:chanrobles.com.ph
Constitution and laws, to wit, ’to conserve and develop the patrimony of the nation.’ as
solemnly enunciated in the preamble to the Constitution. "The nationalization of the natural resources of the country was intended (1) to insure their
conservation for Filipino posterity; (2) to serve as an instrument of national defense,
"‘A narrow and literal interpretation of the phrase ’private agricultural land’ would impair helping prevent the extension into the country of foreign control through peaceful economic
and defeat the nationalistic aim and general policy of our laws and would allow a gradual, penetration; and (3) to prevent making the Philippines a source of international conflicts
steady, and unlimited accumulation in alien hands of a substantial portion of our with the consequent danger to its internal security and independence.
patrimonial estate, to the detriment of our national solidarity, stability and independence.
Nothing could prevent the acquisition of a great portion or the whole of a city by subjects . . .
of a foreign power. And yet a city or urban area is more strategical than a farm or rural
land. ". . . In the preface to its report, the committee on naturalization and preservation of lands
and other natural resources said:jgc:chanrobles.com.ph
"‘The mere literal construction of a section in a statute ought not to prevail if it is opossed
to the intention of the legislature apparent by the statute; and if the words are sufficiently "‘International complications have often resulted from the existence of alien ownership of
flexible to admit of some other construction it is to be adopted to effectuate that intention. land and natural resources in a weak country. Because of this danger, it is best that aliens
The intent prevails over the letter, and the letter will, if possible, be so read as to conform should be restricted in the acquisition of land and other natural resources. An example is
to the spirit of the act. While the intention of the legislature must be ascertained from the afforded by the case of Texas. This state was originally a province of Mexico. In order to
words used to express it, the manifest reason and the obvious purpose of the law should secure its rapid settlement and development, the Mexican government offered free land to
not be sacrificed to a liberal interpretation of such words.’ (II Sutherland, Stat. Construction, settlers in Texas. Americans responded more rapidly than the Mexicans, and soon they
pp. 721, 722.) organized a revolt against Mexican rule,and then secured annexation to the United States.
A new increase of alien landholding in Mexico has brought about a desire to prevent a
"We conclude, therefore, that the residential lot which the applicant seeks to register in his repetition of the Texas affair. Accordingly the Mexican constitution of 1917 contains serious
name falls within the meaning of private agricultural land as this phrase is used in our limitations on the right of aliens to hold lands and mines in Mexico. The Filipinos should
Constitution and, consequently, is not subject to acquisition by foreigners except by profit from this example.’
hereditary succession."cralaw virtua1aw library
. . .
The argument holds water. It expresses a correct interpretation of the Constitution and the
real intent of the Constitutional Convention. "It was primarily for these reasons that the Convention approved readily the proposed
principle of prohibiting aliens to acquire, exploit, develop, or utilize agricultural, timber, and
One of our fellow members therein. Delegate Montilla, said:jgc:chanrobles.com.ph mineral lands of the public domain, waters, minerals, coal, petroleum, and other mineral
oils, all forces of potential energy, and other natural resources of the Philippines. For the
"The constitutional precepts that I believe will ultimately lead us to or desired goal are: (1) same reasons the Convention approved equally readily the proposed principle of
the complete nationalization of our lands and natural resources; (2) the nationalization of prohibiting the transfer or assignment to aliens of private agricultural land, save in case of
our commerce and industry compatible with good international practices. With the hereditary succession." (2 Aruego, Framing of the Philippine Constitution, pp. 604, 605,
complete nationalization of our lands and natural resources it is to be understood that our 606.) .
God-given birthright should be one hundred per cent in Filipino hands. . . . Lands and
natural resources are immovable and as such can be compared to the vital organs of a All the foregoing show why we, having been a member of the Constitutional Convention,
persons’s body, the lack of possession of which may cause instant death or the shortening agree with the Solicitor General’s position and concur in the result of the case, although
of life. If we do not completely nationalize these two or our most important belongings, I we go as far as the outright pronouncement that the purchase made by appellee is null
am afraid that the time will come when we shall be sorry for the time we were born. Our and void.
independence will be just a mockery, for what kind of independence are we going to have
if a part of our country is not in our hands but in those of foreigners?" (2 Aruego, The BRIONES, M., con quien estan conformes PARAS y TUASON, MM.,
Framing of the Philippine Constitution, p. 592.) disidente:chanrob1es virtual 1aw library
6|Page
NAT RES & ENVI LAW CASES 2A_2023
plantea ninguna cuestion de hecho; plantea solo una cuestion de derecho. Por eso que
El solicitante en este expediente pide el registro del solar de que se trata como terreno de en la reconstitucion de este expediente — el original se quemo durante la guerra — no ha
propiedad privada, y tan solo con caracter supletorio invoca las disposiciones del capitulo habido necesidad de incluir las notas taquigraficas ni las pruebas documentales, y de
8. de la Ley No. 2874 sobre terrenos publicos (Pieza de Excepciones, pag. 3.) hecho hemos considerado y decidido este asunto sin dichas notas y pruebas. El abogado
Constantino, del apelado, en la audiencia para la reconstitucion de los autos, hizo esta
Por su parte, el Director de Terrenos se opone a la solicitud en virtud de tres fundamentos, manifestacion: "In view also of the fact that the questions involved here are only questions
a saber: (1) porque ni el solicitante ni sus predecesores en interes pueden demonstrar of law, this representation waives the right to present the evidence presented in the trial
titulo suficiente sobre dicha parcela de terreno, no habiendose adquirido la misma ni port court . . . ." Por su parte, el Procurador General, al explanar el caso en representacion del
titulo de composicion con el Estado bajo la soberania de Espana, ni por titulo de apelante Director de Terrenos, principia su alegato con la siguiente
informacion posesoria bajo el Real Decreto de 13 de Febrero de 1894; (2) porque el citado declaracion:jgc:chanrobles.com.ph
solar es una porcion de los terrenos de dominio publico pertenecientes al Commonwealth
de Filipinas; (3) porque siendo el solicitante un ciudadano chino, no esta capacitado bajo "This appeal is a test case. There are now several cases of exactly the same nature
las disposiciones de la Constitucion de Filipinas para adquirir terrenos de caracter publico pending in the trial courts.
o privado (idem, pags. 5 y 6).
"Whether or not an alien can acquire a residential lot and register it in his name is the only
Tanto el solicitante como el Director de Terrenos practicaron sus pruebas ante un arbitro question raised in this appeal from a decision of the Court of First Instance of Tayabas
nombrado por el Juzgado de Primera Instancia de Tayabas. Con vista de tales pruebas, which sustained the affirmative and decreed the registration of the said property in favor
el Juez Magsalin, del referido Juzgado, dicto sentencia a favor del solicitante, de la cual of the applicant who, by his own voluntary adimission, is a citizen of the Chinese Republic.
transcribimos las siguientes porciones pertinentes:jgc:chanrobles.com.ph This question is raised in connection with the constitutional provision that no private
agricultural land shall be transferred or assigned to foreigners except in cases of hereditary
"La represetnaciond el opositor Director de Terrenos trato de probar por medio del succession." (Pags. 1, 2, alegato del apelante.)
testimonio del Inspector del Buro de Terrenos que el terreno objeto de la solicitud es parte
del dominio publico y ademas el solicitante es ciudadano chino, pero dicho testigo afirmo Habiendose apelado de la sentencia para ante el Tribunal de Apelacion
que el terreno objeto de la presente solicitud es un solar situado dentro de la problacion
del municipio de Guinayangan, Tayabas, y en el mismo existe una casa de materiales
fuertes y careciendo de merito esta oposicion debe desestimarse la misma.

"Por tanto, previa desestimacion de la oposicion del Director de Terrenos, se adjudica con
sus mejoras la parcela de terreno objeto de la presente solicitud descrito en el plano Psu-
109117, a favor del solicitante Oh Cho, ciudadano chino, mayor de edad, casado con Yee
Shi, y residente en el municipio de Guinayangan, Tayabas, Islas Filipinas." (Decision, pag.
8, Record on Appeal.)

De lo transcrito se infiere de una manera forzosa lo siguiente: (a) que el tribunal inferior
desestimo de plano la oposicion del Director de Terrenos fundada en el supuesto de que
el solar cuestionado es parte del dominio publico; (b) que el mismo tribunal rechazo el otro
fundamento de la oposicion, esto es, que siendo el solicitante ciudadano chino esta
incapacitado bajo nuestra Constitucion para adquirir terreno, ya publico, ya privado,
aunque sea un solar de caracter urbano; (c) que, segun el fallo del Juez a quo, no siendo
publico el terreno cuestionado, es necesariamente terreno privado.

El Director de Terrenos, no estando conforme con la sentencia, apelo de ella para ante el
Tribunal de Apelacion y hace en su alegato dos senalamientos de error, ninguno de los
cuales pone en tela de juicio la calidad de privado del terreno cuestionado. El apelante no
7|Page
NAT RES & ENVI LAW CASES 2A_2023
2. Cabuyao Cadastre; and that the same is agricultural in nature and the
improvements found thereon are sugarcane, bamboo clumps, chico and mango
G.R. No. 112567 February 7, 2000 trees and one house of the tenant made of light materials;

THE DIRECTOR, LANDS MANAGEMENT BUREAU, petitioner, 2. That the land subject for registration is outside any civil or military reservation,
vs. riverbed, park and watershed reservation and that same land is free from claim
COURT OF APPEALS and AQUILINO L. CARIÑO, respondents. and conflict;

PURISIMA, J.: 3. That said land is neither inside the relocation site earmarked for Metro Manila
squatters nor any pasture lease; it is not covered by any existing public land
At bar is a Petition for Review on Certiorari under Rule 45 of the Rules of Court, seeking application and no patent or title has been issued therefor;
to set aside the Decision of the Court of Appeals, dated November 11, 1993, in CA-G.R.
No. 29218, which affirmed the Decision, dated February 5, 1990, of Branch XXIV, Regional 4. That the herein petitioner has been in continuous, open and exclusive
Trial Court of Laguna, in LRC No. B-467, ordering the registration of Lot No. 6 in the name possession of the land who acquired the same thru inheritance from his deceased
of the private respondent. mother, Teresa Lauchangco as mentioned on the Extra-judicial partition dated July
26, 1963 which applicant requested that said instrument will be presented on the
The facts that matter are as follows: hearing of this case; and that said land is also declared for taxation purposes under
Tax Declaration No. 6359 in the name of the petitioner;
On May 15, 1975, the private respondent, Aquilino Cariño, filed with the then Branch I,
Court of First Instance of Laguna, a petition1 for registration of Lot No. 6, a sugar land with xxx xxx x x x5
an area of forty-three thousand six hundred fourteen (43,614) square meters, more or less,
forming part of a bigger tract of land surveyed as Psu-108952 and situated in Barrio Sala, With the private respondent as lone witness for his petition, and the Director of Lands as
Cabuyao, Laguna. the only oppositor, the proceedings below ended. On February 5, 1990, on the basis of
the evidence on record, the trial court granted private respondent's petition, disposing thus:
Private respondent declared that subject land was originally owned by his mother, Teresa
Lauchangco, who died on February 15, 1911,2 and later administered by him in behalf of WHEREFORE, the Count hereby orders and declares the registration and
his five brothers and sisters, after the death of their father in 1934.3 confirmation of title to one (1) parcel of land identified as Lot 6, plan Psu-108952,
identical to Cadastral Lot No. 3015, Cad. 455-D, Cabuyao Cadastre, situated in
In 1949, private respondent and his brother, Severino Cariño, became co-owners of Lot the barrio of Sala, municipality of Cabuyao, province of Laguna, containing an area
No. 6 by virtue of an extra-judicial partition of the land embraced in Plan Psu-108952, of FORTY THREE THOUSAND SIX HUNDRED FOURTEEN (43,614) Square
among the heirs of Teresa Lauchangco. On July 26, 1963, through another deed of Meters, more or less, in favor of applicant AQUILINO L. CARINO, married to
extrajudicial settlement, sole ownership of Lot No. 6 was adjudicated to the private Francisca Alomia, of legal age, Filipino, with residence and postal address at Biñan,
respondent.4 Laguna.

Pertinent report of the Land Investigator of the Bureau of Lands (now Bureau of Lands After this decision shall have become final, let an order for the issuance of decree
Management), disclosed: of registration be issued.

xxx xxx xxx SO ORDERED.6

1. That the land subject for registration thru judicial confirmation of imperfect title From the aforesaid decision, petitioner (as oppositor) went to the Court of Appeals, which,
is situated in the barrio of Sala, municipality of Cabuyao, province of Laguna as on November 11, 1993, affirmed the decision appealed from.
described on plan Psu-108952 and is identical to Lot No. 3015, Cad. 455-0,
8|Page
NAT RES & ENVI LAW CASES 2A_2023
Undaunted, petitioner found his way to this Court via the present Petition; theorizing that: Sec. 48. The following described citizens of the Philippines, occupying lands of the
public domain or claiming to own any such lands or an interest therein, but whose
I titles have not been perfected or completed, may apply to the Court of First
Instance of the province where the land is located for confirmation of their claim
THE COURT OF APPEALS ERRED IN NOT FINDING THAT PRIVATE and the issuance of title therefor, under the Land Registration Act, to wit:
RESPONDENT HAS NOT SUBMITTED PROOF OF HIS FEE SIMPLE TITLE OR
PROOF OF POSSESSION IN THE MANNER AND FOR THE LENGTH OF TIME xxx xxx xxx
REQUIRED BY THE LAW TO JUSTIFY CONFIRMATION OF AN IMPERFECT
TITLE. (b) Those who by themselves or through their predecessors-in-interest have been
in open, continuous, exclusive, and notorious possession and occupation of
II agricultural lands of the public domain, under a bona fide claim of acquisition or
ownership, for at least thirty years immediately preceding the filing of the
THE COURT OF APPEALS ERRED IN NOT DECLARING THAT PRIVATE application for confirmation of title except when prevented by war or force majeure.
RESPONDENT HAS NOT OVERTHROWN THE PRESUMPTION THAT THE These shall be conclusively presumed to have performed all the conditions
LAND IS A PORTION OF THE PUBLIC DOMAIN BELONGING TO THE essential to a Government grant and shall be entitled to a certificate of title under
REPUBLIC OF THE PHILIPPINES.7 the provisions of this chapter. (Emphasis supplied)

The Petition is impressed with merit. Possession of public lands, however long, never confers title upon the possessor, unless
the occupant can prove possession or occupation of the same under claim of ownership
for the required period to constitute a grant from the State.13
The petition for land registration8 at bar is under the Land Registration Act.9 Pursuant to
said Act, he who alleges in his petition or application, ownership in fee simple, must present
muniments of title since the Spanish times, such as a titulo real or royal grant, a Notwithstanding absence of opposition from the government, the petitioner in land
concession especial or special grant, a composicion con al estado or adjustment title, or registration cases is not relieved of the burden of proving the imperfect right or title sought
a titulo de compra or title through purchase; and "informacion possessoria" or "possessory to be confirmed. In Director of Lands vs. Agustin,14 this Court stressed that:
information title", which would become a "titulo gratuito" or a gratuitous title.10
. . . The petitioner is not necessarily entitled to have the land registered under the
In the case under consideration, the private respondents (petitioner below) has not Torrens system simply because no one appears to oppose his title and to oppose
produced a single muniment of title substantiate his claim of ownership. 11 The Court has the registration of his land. He must show, even though there is no opposition, to
therefore no other recourse, but to dismiss private respondent's petition for the registration the satisfaction of the court, that he is the absolute owner, in fee simple. Courts
of subject land under Act 496. are not justified in registering property under the Torrens system, simply because
there is no opposition offered. Courts may, even in the absence of any opposition,
deny the registration of the land under the Torrens system, upon the ground that
Anyway, even if considered as petition for confirmation of imperfect title under the Public
the facts presented did not show that petitioner is the owner, in fee simple, of the
land Act (CA No. 141), as amended, private respondent's petition would meet the same
land which he is attempting to have registered.15
fate. For insufficiency of evidence, its denial is inevitable. The evidence adduced by the
private respondent is not enough to prove his possession of subject lot in concept of owner,
in the manner and for the number of years required by law for the confirmation of imperfect There is thus an imperative necessity of the most rigorous scrutiny before imperfect titles
title. over public agricultural lands may be granted judicial
recognition.16
Sec. 48(b) of Commonwealth Act No. 141,12 as amended R.A. No. 1942 and R.A. No.
3872, the law prevailing at the time the Petition of private respondent was filed on May 15, The underlying principle is that all lands that were not acquired from the government, either
1975, provides: by purchase or by grant, belong to the state as part of the public domain. As enunciated
in Republic vs. Lee:17

9|Page
NAT RES & ENVI LAW CASES 2A_2023
. . . Both under the 1935 and the present Constitutions, the conservation no less appear to have any sustainable basis. Said Exhibit "E" shows that it is Tax Declaration
than the utilization of the natural resources is ordained. There would be a failure 1921 for Lot No. 6 in the name of private respondent and not in the name of his parents.22
to abide by its command if the judiciary does not scrutinize with care applications
to private ownership of real estate. To be granted, they must be grounded in well- The rule that findings of fact by the trial court and the Court of Appeals are binding upon
nigh incontrovertible evidence. Where, as in this case, no such proof would be this Court is not without exceptions. Where, as in this case, pertinent records belie the
forthcoming, there is no justification for viewing such claim with favor. It is a basic findings by the lower courts that subject land was declared for taxation purposes in the
assumption of our polity that lands of whatever classification belong to the state. name of private respondent's predecessor-in-interest, such findings have to be
Unless alienated in accordance with law, it retains its right over the same as disregarded by this Court. In Republic vs. Court of Appeals,23 the Court ratiocinated thus:
dominus. . . .18
This case represents an instance where the findings of the lower court overlooked
In order that a petition for registration of land may prosper and the petitioners may savor certain facts of substance and value that if considered would affect the result of
the benefit resulting from the issuance of certificate of title for the land petitioned for, the the case (People v. Royeras, 130 SCRA 259) and when it appears that the
burden is upon him (petitioner) to show that he and/or his predecessor-in-interest has been appellate court based its judgment on a misapprehension of facts (Carolina
in open, continuous, exclusive, and adverse possession and occupation of the land sought Industries, Inc. v. CMS Stock Brokerage, Inc., et al., 97 SCRA 734; Moran, Jr. v.
for registration, for at least (30) thirty years immediately preceding the filing of the petition Court of Appeals, 133 SCRA 88; Director of Lands v. Funtillar, et al., G.R. No.
for confirmation of title.19 68533, May 3, 1986). This case therefore is an exception to the general rule that
the findings of facts of the Court of Appeals are final and conclusive and cannot be
In the case under consideration, private respondent can only trace his own possession of reviewed on appeal to this Court.'
subject parcel of land to the year 1949, when the same was adjudicated to him by virtue
of an extra-judicial settlement and partition. Assuming that such a partition was truly and—
effected, the private respondent has possessed the property thus partitioned for only
twenty-six (26) years as of 1975, when he filed his petition for the registration thereof. To . . . in the interest of substantial justice this Court is not prevented from considering
bridge the gap, he proceeded to tack his possession to what he theorized upon as such a pivotal factual matter that had been overlooked by the Courts below. The
possession of the same land by his parents. However, other than his unilateral assertion, Supreme Court is clothed with ample authority to review palpable errors not
private respondent has not introduced sufficient evidence to substantiate his allegation assigned as such if it finds that their consideration is necessary in arriving at a just
that his late mother possessed the land in question even prior to 1911.1âwphi1.nêt decision.24

Basic is the rule that the petitioner in a land registration case must prove the facts and Verily, the Court of Appeals just adopted entirely the findings of the trial court. Had it
circumstances evidencing his alleged ownership of the land applied for. General examined the original records of the case, the said court could have verified that the land
statements, which are mere conclusions of law and not factual proof of possession are involved was never declared for taxation purposes by the parents of the respondent. Tax
unavailing and cannot suffice.20 receipts and tax declarations are not incontrovertible evidence of ownership. They are
mere indicia of claim of ownership.25 In Director of Lands vs. Santiago.26
From the relevant documentary evidence, it can be gleaned that the earliest tax declaration
covering Lot No. 6 was Tax Declaration No. 3214 issued in 1949 under the names of the . . . if it is true that the original owner and possessor, Generosa Santiago, had been
private respondent and his brother, Severino Cariño. The same was followed by Tax in possession since 1925, why were the subject lands declared for taxation
Declaration No. 1921 issued in 1969 declaring an assessed value of Five Thousand Two purposes for the first time only in 1968, and in the names of Garcia and Obdin?
Hundred Thirty-three (P5,233.00) Pesos and Tax Declaration No. 6359 issued in 1974 in For although tax receipts and declarations of ownership for taxation purposes are
the name of private respondent, declaring an assessment of Twenty-One Thousand Seven not incontrovertible evidence of ownership, they constitute at least proof that the
Hundred Seventy (P21,770.00) Pesos.21 holder had a claim of title over the property.27

It bears stressing that the Exhibit "E" referred to in the decision below as the tax declaration As stressed by the Solicitor General, the contention of private respondent that his mother
for subject land under the names of the parents of herein private respondent does not had been in possession of subject land even prior to 1911 is self-serving, hearsay, and
inadmissible in evidence. The phrase "adverse, continuous, open, public, and in concept
10 | P a g e
NAT RES & ENVI LAW CASES 2A_2023
of owner", by which characteristics private respondent describes his possession and that Neither can private respondent seek refuge under P.D. No. 1073,33 amending Section 48(b)
of his parents, are mere conclusions of law requiring evidentiary support and substantiation. of Commonwealth Act No. 141 under which law a certificate of title may issue to any
The burden of proof is on the private respondent, as applicant, to prove by clear, positive occupant of a public land, who is a Filipino citizen, upon proof of open, continuous
and convincing evidence that the alleged possession of his parents was of the nature and exclusive, and notorious possession and occupation since June 12, 1945, or earlier.
duration required by law. His bare allegations without more, do not amount to preponderant Failing to prove that his predecessors-in-interest occupied subject land under the
evidence that would shift the burden of proof to the oppositor.28 conditions laid down by law, the private respondent could only establish his possession
since 1949, four years later than June 12, 1945, as set by law.
In a case,29 this Court set aside the decisions of the trial court and the Court of Appeals
for the registration of a parcel of land in the name of the applicant, pursuant to Section 48 The Court cannot apply here the juris et de jure presumption that the lot being claimed by
(b) of the Public Land Law; holding as follows: the private respondent ceased to be a public land and has become private property. 34 To
reiterate, under the Regalian doctrine all lands belong to the State.35 Unless alienated in
Based on the foregoing, it is incumbent upon private respondent to prove that the accordance with law, it retains its basic rights over the same as dominus.36
alleged twenty year or more possession of the spouses Urbano Diaz and Bernarda
Vinluan which supposedly formed part of the thirty (30) year period prior to the Private respondent having failed to come forward with muniments of title to reinforce his
filing of the application, was open, continuous, exclusive, notorious and in concept petition for registration under the Land Registration Act (Act 496), and to present
of owners. This burden, private respondent failed to discharge to the satisfaction convincing and positive proof of his open, continuous, exclusive and notorious occupation
of the Court. The bare assertion that the spouses Urbano Diaz and Bernarda of Lot No. 6 en concepto de dueño for at least 30 years immediately preceding the filing of
Vinluan had been in possession of the property for more than twenty (20) years his petition,37 the Court is of the opinion, and so finds, that subject Lot No. 6 surveyed
found in private respondent's declaration is hardly the "well-nigh incontrovertible" under Psu-108952, forms part of the public domain not registrable in the name of private
evidence required in cases of this nature. Private respondent should have respondent.
presented specific facts that would have shown the nature of such
possession. . . .30 WHEREFORE, the Petition is GRANTED; the Decision of the Court of Appeals, dated
November 11, 1993, in CA-G.R. No. 29218 affirming the Decision, dated February 5, 1990,
In Director of Lands vs. Datu,31 the application for confirmation of imperfect title was of Branch XXIV, Regional Trial Court of Laguna in LRC No. 8-467, is SET ASIDE; and Lot
likewise denied on the basis of the following disquisition, to wit: No. 6, covered by and more particularly described in Psu-108952, is hereby declared a
public land, under the administrative supervision and power of disposition of the Bureau of
We hold that applicants' nebulous evidence does not support their claim of open, Lands Management. No pronouncement as to costs.1âwphi1.nêt
continuous, exclusive and notorious occupation of Lot No. 2027-B en concepto de
dueño. Although they claimed that they have possessed the land since 1950, they SO ORDERED.
declared it for tax purposes only in 1972. It is not clear whether at the time they
filed their application in 1973, the lot was still cogon land or already cultivated land.

They did not present as witness their predecessor, Peñaflor, to testify on his
alleged possession of the land. They alleged in their application that they had
tenants on the land. Not a single tenant was presented as witness to prove that
the applicants had possessed the land as owners.

xxx xxx xxx

On the basis of applicants' insubstantial evidence, it cannot justifiably be


concluded that they have an imperfect title that should be confirmed or that they
had performed all the conditions essential to a Government grant of a portion of
the public domain.32
11 | P a g e
NAT RES & ENVI LAW CASES 2A_2023
3. availed of by the applicant because it failed to file an appropriate application for registration
in accordance with the provisions of Presidential Decree (P.D.) No. 892; and that the
G.R. No. 150000 September 26, 2006 subject parcels of land are portions of the public domain belonging to the Republic of the
Philippines and are not subject to private appropriation.7
REPUBLIC OF THE PHILIPPINES, petitioner,
vs. On September 19, 1997, Tri-Plus presented documentary evidence to prove compliance
TRI-PLUS CORPORATION, respondent. with the jurisdictional requirements of the law. On even date, a Manifestation and Motion
was filed by the heirs of Toribio Pepito praying that they be given a period of 10 days within
DECISION which to file their written opposition.8 However, the oppositors failed to file their written
opposition on time. The trial court then commissioned its clerk of court to receive evidence
from the applicant and directed the former to submit a report thereon. Accordingly, a
AUSTRIA-MARTINEZ, J.:
Commissioner's Report was submitted on the proceedings taken.9
Before the Court is a petition for review on certiorari under Rule 45 of the Rules of Court
In its Judgment dated February 26, 1998, the MTC made the following finding and
assailing the Decision1 dated September 14, 2001 of the Court of Appeals (CA) in CA-G.R.
conclusion:
CV No. 60671, which affirmed the judgment of the Municipal Trial Court (MTC) of
Consolacion, Metro Cebu in LRC Case No. N-21 granting herein respondent's application
for registration of title to Lots Nos. 1061 and 1062 of the Cadastral Survey of Consolacion, The totality of the evidence, both documentary and testimonial, of the applicant
Cebu. clearly shows that it and its predecessors-in-interest had been in actual, public,
exclusive and continuous possession in concept of owner of the parcels of land
above-mentioned for no less than thirty (30) years prior to the filing of the instant
The facts of the case are as follows:
petition for registration of its imperfect title. This being so, the applicant is entitled
that its title be confirmed under the provisions of the Torrens System of
On April 30, 1997 Tri-Plus Corporation2, through its president, Euclid C. Po, filed with the Registration.10
MTC of Consolacion, Metro Cebu,3 an Application for Registration of Title over two parcels
of land designated as Lots 1061 and 1062 of the cadastral survey of Consolacion, Cebu,
Accordingly, it disposed of the case as follows:
containing an area of 3,939 and 4,796 square meters, respectively, and located at
Barangay Tayud, Consolacion, Cebu.4 In its application, Tri-Plus alleged that it is the
owner in fee simple of the subject parcels of land, including the improvements thereon, WHEREFORE, in view of the foregoing, judgment is hereby rendered declaring
having acquired the same through purchase; and that it is in actual, continuous, public, the applicant TRI-PLUS LAND CORPORATION the exclusive and absolute owner
notorious, exclusive and peaceful possession of the subject properties in the concept of of Lot 1061 of the Cadastral Survey of Consolacion, Cebu, as shown on plan Ap-
an owner for more than 30 years, including that of its predecessors-in-interest.5 The case 07-002362 (Exhibit "J") and described in its corresponding technical description
was docketed as LRC Case No. N-21.6 (Exhibit "K"), and Lot 1062 of the Cadastral Survey of Consolacion, Cebu, as
shown on plan Ap-07-002366 (Exhibit "O") and described in its corresponding
technical description (Exhibit "P").
On September 4, 1997, the trial court received an Opposition to the Application for
Registration filed by the Republic of the Philippines through the Office of the Solicitor
General (OSG) on the grounds that neither the applicant nor its predecessors-in-interest Once this decision becomes final, let an Order for the issuance of the decree of
have been in open, continuous, exclusive and notorious possession and occupation of the registration for Lots 1061 and 1062, Consolacion Cadastre, be issued in the name
land in question since June 12, 1945 or prior thereto; that the muniments of title submitted of TRI-PLUS LAND CORPORATION.
by the applicant which consists, among others, of tax declarations and receipts of tax
payments, do not constitute competent and sufficient evidence of a bona fide acquisition SO ORDERED.11
of the land applied for or of its open, continuous, exclusive and notorious possession and
occupation thereof in the concept of owner since June 12, 1945 or prior thereto; that the The OSG appealed the trial court's judgment with the CA.12
claim of ownership in fee simple on the basis of a Spanish title or grant may no longer be

12 | P a g e
NAT RES & ENVI LAW CASES 2A_2023
Subsequently, the Land Registration Authority (LRA), through its Director on Registration, THE COURT OF APPEALS COMMITTED REVERSIBLE ERROR IN NOT
submitted a Report dated August 6, 1998 to the MTC, pertinent portions of which read as FINDING THAT RESPONDENT IS DISQUALIFIED FROM ACQUIRING LANDS
follows: OF THE PUBLIC DOMAIN.15

1. Two (2) parcels of land described as Lots 1062 and 1061, Cad. 545-D, As to the first assigned error, petitioner contends that the CA erred in relying on the original
Consolacion Cadastre on Plan Ap-07-002366 and Ap-07-002362, both situated in survey plan approved by the Lands Management Services of the Department of
the Barangay of Tayud, Municipality of Consolacion, Province of Cebu, are being Environment and Natural Resources (DENR) when it ruled that the applicant was able to
applied for original registration of title; duly establish the identity of Lot 1061. This reliance, petitioner argues, is mistaken
considering that the Report of the Director on Registration of the LRA pointed to a
2. After examining the afore-said plan discrepancy was noted in the bearings and discrepancy in the bearings and distances of the boundaries which separate Lot 1061 from
distances of line 3-4 and 4-5 of Lot 1061, Ap-07-002362, being S.57 deg. 19'W an adjoining land, Lot 1058. This discrepancy, petitioners submit, casts doubt on the
8.02m. and S.52 deg. 10'W 18.24, which do not conform with the bearings and identity of the land subject of the application for registration. Petitioner then concludes that
distances (N. 52 deg. 01'E., 18.00m) and (N. 52 deg. 47'E., 17.71m.) along lines if there is uncertainty in the metes and bounds of the property sought to be titled, the trial
12-13 and 11-12, respectively of plan Rs-07-01-000358, lot 1508, Consolacion court cannot acquire jurisdiction over the subject matter of the case. Hence, the
Cad. 545-D, decreed in LRA (NALTDRA) Record No. N-60851. proceedings before the trial court, including its decision granting the application for
registration, are void.
3. That the above discrepancy was brought to the attention of the Regional
Technical Director, DENR, Land Management Services, Region VII, Mandaue City, As to the second assignment of error, petitioner argues that the CA erred in holding that
for verification and correction in a letter dated 7 July 1998. the applicant was able to prove that the subject properties are alienable and disposable
lands of the public domain. Petitioner contends that a mere notation appearing in the
4. This Authority is not in a position to verify whether or not the parcels of land survey plans of the disputed properties showing that the subject lands had been classified
subject of registration are already covered by land patent.13 as alienable and disposable on June 25, 1963 is not sufficient to establish the nature and
character of these lands. Petitioner asserts that there should be a positive act on the part
of the government, such as a certification from the DENR, to prove that the said lands are
On September 14, 2001, the CA rendered the presently assailed Decision finding no
indeed alienable and disposable. Petitioner further contends that even if the subject
reversible error in the appealed judgment, thereby, affirming the same.14
properties were classified as alienable and disposable on June 25, 1963, the law,
nonetheless, requires that such classification should have been made on June 12, 1945
Hence, herein petition based on the following assignments of errors: or earlier.

I Anent the last assigned error, petitioner contends that since the applicant failed to
discharge the burden of proving that the subject properties are alienable and disposable,
THE COURT OF APPEALS COMMITTED REVERSIBLE ERROR IN NOT there is no basis for the CA to rule that these properties are private lands.
FINDING THAT THE TRIAL COURT DID NOT ACQUIRE JURISDICTION TO
HEAR AND DECIDE THE CASE, BECAUSE THE IDENTITY OF THE LAND In its Comment, respondent contends that it was able to prove the identity of Lot 1061 with
REMAINS UNCERTAIN. certainty. While it admits the discrepancy in the bearings and distances which form the
boundary between Lot 1061 and the adjoining Lot 1058, respondent contends that such
II discrepancy is merely technical in nature because Lots 1058 and 1061 remain the same
and that there is neither an increase nor decrease in the area of the subject lot sought to
THE COURT OF APPEALS COMMITTED REVERSIBLE ERROR IN NOT be titled; and that what was required by the LRA in its Report was for the applicant to
FINDING THAT RESPONDENT FAILED TO DISCHARGE THE BURDEN OF correct and adjust the bearings and distances of Lot 1061 in order to conform to the
PROVING THAT THE PROPERTY IS ALIENABLE AND DISPOSABLE. boundaries of Lot 1058.

III
13 | P a g e
NAT RES & ENVI LAW CASES 2A_2023
Respondent also argues that the notations appearing in the survey plans of the subject of the subject lot and at the same time prejudice the adjoining lot owner. It is only when
properties serve as sufficient proof that these lands are alienable and disposable. the discrepancy results to an unexplained increase in the total area of the land sought to
Respondent asserts that the survey plans were duly approved by the DENR, Lands be registered that its identity is made doubtful. Besides, only a portion of the many
Management Services whose official acts are presumed to be in accordance with law. boundaries of Lot 1061 has been found to bear a discrepancy in relation to the boundary
of one adjoining lot and the LRA Report simply recommends that the Lands Management
Lastly, respondent argues that its predecessor-in-interest's continuous, actual, adverse Services of the DENR verify the reported discrepancy and make the necessary corrections,
and peaceful possession of the subject properties in the concept of an owner for a period if needed, in order to avoid duplication in the issuance of titles covering the same parcels
of more than 30 years, coupled with the fact that they declared these lands in their name, of land.
gives a strong presumption in respondent's favor that the subject properties no longer form
part of the public domain. Petitioner's argument that, on the basis of the LRA Report, the MTC should have dismissed
respondent's application for registration for lack of jurisdiction over the subject matter, is
Parties filed their respective Memoranda.16 without merit. The MTC could not have possibly done this because said Report was
submitted to the trial court more than five months after the latter rendered its Decision. A
The Court finds the petition meritorious. copy of the LRA Report attached to the present petition shows that it is dated August 6,
1998 while the MTC decision was rendered much earlier on February 26, 1998. In fact,
the Office of the Solicitor General (OSG) perfected its appeal by filing a notice of appeal
At the outset, however, the Court does not agree with petitioner's contention in its first
of the MTC Decision on April 2, 1998, which is also prior to the submission of the LRA
assigned error that respondent failed to properly identify Lot 1061 which is one of the lots
report. Hence, by the time the LRA report was submitted to the MTC, the latter has already
sought to be titled.
lost jurisdiction over the case, not on the ground cited by petitioner but because the appeal
to the CA was already perfected, vesting jurisdiction upon the appellate court.
Insofar as the identity of the land subject of an application for original registration is
concerned, this Court has laid down the rule, as follows:
In any case, while the subject lands were properly identified, the Court finds that
respondent failed to comply with the other legal requirements for its application for
The submission in evidence of the original tracing cloth plan, duly approved by the registration to be granted.
Bureau of Lands, in cases for application of original registration of land is a
mandatory requirement. The reason for this rule is to establish the true identity of
Applicants for confirmation of imperfect title must prove the following: (a) that the land
the land to ensure that it does not overlap a parcel of land or a portion thereof
forms part of the alienable and disposable agricultural lands of the public domain; and (b)
already covered by a previous land registration, and to forestall the possibility that
that they have been in open, continuous, exclusive and notorious possession and
it will be overlapped by a subsequent registration of any adjoining land. The failure
occupation of the same under a bona fide claim of ownership either since time immemorial
to comply with this requirement is fatal to petitioner's application for registration.17
or since June 12, 1945.23
However, in Republic of the Philippines v. Court of Appeals18 and in the more recent cases
In the present case, the Court finds merit in petitioner's contention that respondent failed
of Spouses Recto v. Republic of the Philippines19 and Republic of the Philippines v.
to prove the first requirement that the properties sought to be titled forms part of the
Hubilla20, the Court ruled that while the best evidence to identify a piece of land for
alienable and disposable agricultural lands of the public domain.
registration purposes is the original tracing cloth plan from the Bureau of Lands (now the
Lands Management Services of the DENR), blueprint copies and other evidence could
also provide sufficient identification. In the present case, respondent submitted in evidence Section 6 of Commonwealth Act No. 141, as amended, provides that the classification and
a blueprint copy of the Advance Plan of Lot 106121 and a Technical Description22 thereof, reclassification of public lands into alienable or disposable, mineral or forest land is the
both of which had been duly certified and approved by the Lands Management Services prerogative of the Executive Department. Under the Regalian doctrine, which is embodied
of the DENR. The Court finds these pieces of evidence as substantial compliance with the in our Constitution, all lands of the public domain belong to the State, which is the source
legal requirements for the proper identification of Lot 1061. The discrepancy in the of any asserted right to any ownership of land.24 All lands not appearing to be clearly within
common boundary that separates Lot 1061 from Lot 1058, as contained in the LRA Report private ownership are presumed to belong to the State.25 Accordingly, public lands not
does not cast doubt on the identity of the subject lot. As the CA correctly held, the shown to have been reclassified or released as alienable agricultural land or alienated to
discrepancy is not substantial because it does not unduly increase or affect the total area a private person by the State remain part of the inalienable public domain.26
14 | P a g e
NAT RES & ENVI LAW CASES 2A_2023
It must be stressed that incontrovertible evidence must be presented to establish that the for a property that is not in his actual, or at least, constructive possession.35 In the present
land subject of the application is alienable or disposable.27 case, respondent failed to explain why, despite the claim of its predecessors-in interest
that they possessed the subject properties in the concept of an owner as early as 1947, it
In the present case, the only evidence to prove the character of the subject lands as was only in 1961 that they started to declare the same for purposes of taxation.
required by law is the notation appearing in the Advance Plan stating in effect that the said
properties are alienable and disposable. However, this is hardly the kind of proof required From the foregoing, it is clear that respondent and its predecessors-in-interest failed to
by law. To prove that the land subject of an application for registration is alienable, an prove that they had been in open, continuous, exclusive and notorious possession of the
applicant must establish the existence of a positive act of the government such as a subject properties under a bona fide claim of ownership since June 12, 1945 or earlier, as
presidential proclamation or an executive order, an administrative action, investigation required by law.
reports of Bureau of Lands investigators, and a legislative act or statute. 28 The applicant
may also secure a certification from the Government that the lands applied for are Well-entrenched is the rule that the burden of proof in land registration cases rests on the
alienable and disposable.29 In the case at bar, while the Advance Plan bearing the notation applicant who must show clear, positive and convincing evidence that his alleged
was certified by the Lands Management Services of the DENR, the certification refers only possession and occupation were of the nature and duration required by law. 36 In the
to the technical correctness of the survey plotted in the said plan and has nothing to do present case, the Court finds that respondent failed to prove, by clear and convincing
whatsoever with the nature and character of the property surveyed. Respondents failed to evidence, the legal requirements that the lands sought to be titled are alienable and
submit a certification from the proper government agency to prove that the lands subject disposable and that its predecessors-in-interest were already in possession of the subject
for registration are indeed alienable and disposable. lots since 1945 or earlier.

As to the second requirement, testimonial evidence were presented to prove that As to the last assigned error, respondent having failed to prove that the subject properties
respondent's predecessors-in-interest had been in possession of the subject lots in the are alienable and disposable public lands, the Court agrees with petitioner that there would
concept of an owner for the period required by law. The first witness was Thelma Pilapil be no basis in concluding that these lands have already become private. The presumption
who claims to be the daughter of Constancia Frias from whom respondent bought Lot 1061. remains that said properties remain part of the inalienable public domain and, therefore,
Pilapil testified that her family has been in possession of Lot 1061 since her birth. 30 When could not become the subject of confirmation of imperfect title.
her testimony was offered on October 7, 1997, she was 40 years old.31 Deducting 40 years
from 1997, it means that her family started possession of Lot 1061 only in 1957. The Finally, while it is an acknowledged policy of the State to promote the distribution of
second witness who was presented was Tomas Frias from whom respondent bought Lot alienable public lands as a spur to economic growth and in line with the ideal of social
1062. Frias testified that he was 67 years old at the time that his testimony was taken on justice, the law imposes stringent safeguards upon the grant of such resources lest they
October 7, 1997.32 He claims that he started owning the subject lot when he was 17 years fall into the wrong hands to the prejudice of the national patrimony.37 The Court must not,
old and had been in possession of the same since then.33 Hence, by simple arithmetic, the therefore, relax the stringent safeguards relative to the registration of imperfect titles.
testimony of Frias proves that he came to possess Lot 1062 only in 1947. While he testified
that Lot 1062 was previously owned by his father and that he inherited the property from
WHEREFORE, the instant petition is GRANTED. The Decision of the Court of Appeals
his parents, no evidence was presented to show that the latter indeed previously owned
dated September 14, 2001 in CA-G.R. CV No. 60671 is REVERSED and SET ASIDE.
the said property and that they had been in possession of the same on or before June 12,
Respondent Tri-Plus Corporation's application for registration and issuance of title to Lots
1945.
1061 and 1062, Consolacion Cad-545-D, in LRC Case No. N-21 filed with the Municipal
Trial Court of Consolacion, Metro Cebu, is DISMISSED.
Moreover, other pieces of evidence presented by respondent to prove the period of its
possession and that of its predecessors-in-interest show that the subject properties were
SO ORDERED.
declared for taxation purposes beginning only in 1961.34 This date may be considered as
relatively recent considering that respondent's predecessors-in-interest claim to have been
in possession of the subject properties as early as 1947. While belated declaration of a
property for taxation purposes does not necessarily negate the fact of possession, tax
declarations or realty tax payments of property are, nevertheless, good indicia of
possession in the concept of an owner, for no one in his right mind would be paying taxes

15 | P a g e
NAT RES & ENVI LAW CASES 2A_2023
4. nullification of Proclamation No. 10645">[3] issued by President Gloria Macapagal-Arroyo
classifying Boracay into reserved forest and agricultural land.
G.R. No. 167707 October 8, 2008
The Antecedents
THE SECRETARY OF THE DEPARTMENT OF ENVIRONMENT AND NATURAL
RESOURCES, THE REGIONAL EXECUTIVE DIRECTOR, DENR-REGION VI, G.R. No. 167707
REGIONAL TECHNICAL DIRECTOR FOR LANDS, LANDS MANAGEMENT BUREAU,
REGION VI PROVINCIAL ENVIRONMENT AND NATURAL RESOURCES OFFICER OF Boracay Island in the Municipality of Malay, Aklan, with its powdery white sand beaches
KALIBO, AKLAN, REGISTER OF DEEDS, DIRECTOR OF LAND REGISTRATION and warm crystalline waters, is reputedly a premier Philippine tourist destination. The
AUTHORITY, DEPARTMENT OF TOURISM SECRETARY, DIRECTOR OF PHILIPPINE island is also home to 12,003 inhabitants4 who live in the bone-shaped island’s
TOURISM AUTHORITY, petitioners, three barangays.5
vs.
MAYOR JOSE S. YAP, LIBERTAD TALAPIAN, MILA Y. SUMNDAD, and ANICETO On April 14, 1976, the Department of Environment and Natural Resources (DENR)
YAP, in their behalf and in behalf of all those similarly situated, respondents. approved the National Reservation Survey of Boracay

Island,6 which identified several lots as being occupied or claimed by named persons.7
x - - - - - - - - - - - - - - - - - - - - - - - - - - - - - - - - - - - - - - - - - - - - - - - - - - x
On November 10, 1978, then President Ferdinand Marcos issued Proclamation
No. 18018 declaring Boracay Island, among other islands, caves and peninsulas in the
G.R. No. G.R. No. 173775 October 8, 2008 Philippines, as tourist zones and marine reserves under the administration of the
Philippine Tourism Authority (PTA). President Marcos later approved the issuance of PTA
DR. ORLANDO SACAY and WILFREDO GELITO, joined by THE LANDOWNERS OF Circular 3-829 dated September 3, 1982, to implement Proclamation No. 1801.
BORACAY SIMILARLY SITUATED NAMED IN A LIST, ANNEX "A" OF THIS
PETITION, petitioners, Claiming that Proclamation No. 1801 and PTA Circular No 3-82 precluded them from filing
vs. an application for judicial confirmation of imperfect title or survey of land for titling purposes,
THE SECRETARY OF THE DEPARTMENT OF ENVIRONMENT AND NATURAL respondents-claimants
RESOURCES, THE REGIONAL TECHNICAL DIRECTOR FOR LANDS, LANDS Mayor Jose S. Yap, Jr., Libertad Talapian, Mila Y. Sumndad, and Aniceto Yap filed a
MANAGEMENT BUREAU, REGION VI, PROVINCIAL ENVIRONMENT AND NATURAL petition for declaratory relief with the RTC in Kalibo, Aklan.
RESOURCES OFFICER, KALIBO, AKLAN, respondents.
In their petition, respondents-claimants alleged that Proclamation No. 1801 and PTA
DECISION Circular No. 3-82 raised doubts on their right to secure titles over their occupied lands.
They declared that they themselves, or through their predecessors-in-interest, had been
REYES, R.T., J.: in open, continuous, exclusive, and notorious possession and occupation in Boracay since
June 12, 1945, or earlier since time immemorial. They declared their lands for tax purposes
AT stake in these consolidated cases is the right of the present occupants of Boracay and paid realty taxes on them.10
Island to secure titles over their occupied lands.
Respondents-claimants posited that Proclamation No. 1801 and its implementing Circular
There are two consolidated petitions. The first is G.R. No. 167707, a petition for review did not place Boracay beyond the commerce of man. Since the Island was classified as a
on certiorari of the Decision1 of the Court of Appeals (CA) affirming that2 of the Regional tourist zone, it was susceptible of private ownership. Under Section 48(b) of
Trial Court (RTC) in Kalibo, Aklan, which granted the petition for declaratory relief filed by Commonwealth Act (CA) No. 141, otherwise known as the Public Land Act, they had the
respondents-claimants Mayor Jose Yap, et al. and ordered the survey of Boracay for titling right to have the lots registered in their names through judicial confirmation of imperfect
purposes. The second is G.R. No. 173775, a petition for prohibition, mandamus, and titles.

16 | P a g e
NAT RES & ENVI LAW CASES 2A_2023
The Republic, through the Office of the Solicitor General (OSG), opposed the petition for SO ORDERED.17
declaratory relief. The OSG countered that Boracay Island was an unclassified land of
the public domain. It formed part of the mass of lands classified as "public forest," which The RTC upheld respondents-claimants’ right to have their occupied lands titled in their
was not available for disposition pursuant to Section 3(a) of Presidential Decree (PD) No. name. It ruled that neither Proclamation No. 1801 nor PTA Circular No. 3-82 mentioned
705 or the Revised Forestry Code,11 as amended. that lands in Boracay were inalienable or could not be the subject of disposition.18 The
Circular itself recognized private ownership of lands.19 The trial court cited Sections
The OSG maintained that respondents-claimants’ reliance on PD No. 1801 and PTA 8720 and 5321 of the Public Land Act as basis for acknowledging private ownership of lands
Circular No. 3-82 was misplaced. Their right to judicial confirmation of title was governed in Boracay and that only those forested areas in public lands were declared as part of the
by CA No. 141 and PD No. 705. Since Boracay Island had not been classified as alienable forest reserve.22
and disposable, whatever possession they had cannot ripen into ownership.
The OSG moved for reconsideration but its motion was denied. 23 The Republic then
During pre-trial, respondents-claimants and the OSG stipulated on the following facts: (1) appealed to the CA.
respondents-claimants were presently in possession of parcels of land in Boracay Island;
(2) these parcels of land were planted with coconut trees and other natural growing trees; On December 9, 2004, the appellate court affirmed in toto the RTC decision, disposing as
(3) the coconut trees had heights of more or less twenty (20) meters and were planted follows:
more or less fifty (50) years ago; and (4) respondents-claimants declared the land they
were occupying for tax purposes.12 WHEREFORE, in view of the foregoing premises, judgment is hereby rendered by us
DENYING the appeal filed in this case and AFFIRMING the decision of the lower court.24
The parties also agreed that the principal issue for resolution was purely legal: whether
Proclamation No. 1801 posed any legal hindrance or impediment to the titling of the lands The CA held that respondents-claimants could not be prejudiced by a declaration that the
in Boracay. They decided to forego with the trial and to submit the case for resolution upon lands they occupied since time immemorial were part of a forest reserve.
submission of their respective memoranda.13
Again, the OSG sought reconsideration but it was similarly denied.25 Hence, the present
The RTC took judicial notice14 that certain parcels of land in Boracay Island, more petition under Rule 45.
particularly Lots 1 and 30, Plan PSU-5344, were covered by Original Certificate of Title No.
19502 (RO 2222) in the name of the Heirs of Ciriaco S. Tirol. These lots were involved in
G.R. No. 173775
Civil Case Nos. 5222 and 5262 filed before the RTC of Kalibo, Aklan.15 The titles were
issued on
On May 22, 2006, during the pendency of G.R. No. 167707, President Gloria Macapagal-
Arroyo issued Proclamation No. 106426 classifying Boracay Island into four hundred (400)
August 7, 1933.16
hectares of reserved forest land (protection purposes) and six hundred twenty-eight and
96/100 (628.96) hectares of agricultural land (alienable and disposable). The Proclamation
RTC and CA Dispositions likewise provided for a fifteen-meter buffer zone on each side of the centerline of roads
and trails, reserved for right-of-way and which shall form part of the area reserved for forest
On July 14, 1999, the RTC rendered a decision in favor of respondents-claimants, with land protection purposes.
a fallo reading:
On August 10, 2006, petitioners-claimants Dr. Orlando Sacay,27 Wilfredo Gelito,28 and
WHEREFORE, in view of the foregoing, the Court declares that Proclamation No. 1801 other landowners29 in Boracay filed with this Court an original petition for prohibition,
and PTA Circular No. 3-82 pose no legal obstacle to the petitioners and those similarly mandamus, and nullification of Proclamation No. 1064.30 They allege that the
situated to acquire title to their lands in Boracay, in accordance with the applicable laws Proclamation infringed on their "prior vested rights" over portions of Boracay. They have
and in the manner prescribed therein; and to have their lands surveyed and approved by been in continued possession of their respective lots in Boracay since time immemorial.
respondent Regional Technical Director of Lands as the approved survey does not in itself They have also invested billions of pesos in developing their lands and building
constitute a title to the land. internationally renowned first class resorts on their lots.31

17 | P a g e
NAT RES & ENVI LAW CASES 2A_2023
Petitioners-claimants contended that there is no need for a proclamation reclassifying HAVE PETITIONERS OCCUPANTS ACQUIRED PRIOR VESTED RIGHT OF PRIVATE
Boracay into agricultural land. Being classified as neither mineral nor timber land, the OWNERSHIP OVER THEIR OCCUPIED PORTIONS OF BORACAY LAND, DESPITE
island is deemed agricultural pursuant to the Philippine Bill of 1902 and Act No. 926, THE FACT THAT THEY HAVE NOT APPLIED YET FOR JUDICIAL CONFIRMATION OF
known as the first Public Land Act.32 Thus, their possession in the concept of owner for IMPERFECT TITLE?
the required period entitled them to judicial confirmation of imperfect title.
III.
Opposing the petition, the OSG argued that petitioners-claimants do not have a vested
right over their occupied portions in the island. Boracay is an unclassified public forest land IS THE EXECUTIVE DECLARATION OF THEIR AREAS AS ALIENABLE AND
pursuant to Section 3(a) of PD No. 705. Being public forest, the claimed portions of the DISPOSABLE UNDER SEC 6, CA 141 [AN] INDISPENSABLE PRE-REQUISITE FOR
island are inalienable and cannot be the subject of judicial confirmation of imperfect title. PETITIONERS TO OBTAIN TITLE UNDER THE TORRENS SYSTEM?
It is only the executive department, not the courts, which has authority to reclassify lands
of the public domain into alienable and disposable lands. There is a need for a positive IV.
government act in order to release the lots for disposition.
IS THE ISSUANCE OF PROCLAMATION 1064 ON MAY 22, 2006, VIOLATIVE OF THE
On November 21, 2006, this Court ordered the consolidation of the two petitions as they PRIOR VESTED RIGHTS TO PRIVATE OWNERSHIP OF PETITIONERS OVER THEIR
principally involve the same issues on the land classification of Boracay Island.33 LANDS IN BORACAY, PROTECTED BY THE DUE PROCESS CLAUSE OF THE
CONSTITUTION OR IS PROCLAMATION 1064 CONTRARY TO SEC. 8, CA 141, OR
Issues SEC. 4(a) OF RA 6657.

G.R. No. 167707 V.

The OSG raises the lone issue of whether Proclamation No. 1801 and PTA Circular No. CAN RESPONDENTS BE COMPELLED BY MANDAMUS TO ALLOW THE SURVEY
3-82 pose any legal obstacle for respondents, and all those similarly situated, to acquire AND TO APPROVE THE SURVEY PLANS FOR PURPOSES OF THE APPLICATION
title to their occupied lands in Boracay Island.34 FOR TITLING OF THE LANDS OF PETITIONERS IN BORACAY?35 (Underscoring
supplied)
G.R. No. 173775
In capsule, the main issue is whether private claimants (respondents-claimants in G.R. No.
Petitioners-claimants hoist five (5) issues, namely: 167707 and petitioners-claimants in G.R. No. 173775) have a right to secure titles over
their occupied portions in Boracay. The twin petitions pertain to their right, if any, to judicial
I. confirmation of imperfect title under CA No. 141, as amended. They do not involve their
right to secure title under other pertinent laws.
AT THE TIME OF THE ESTABLISHED POSSESSION OF PETITIONERS IN CONCEPT
OF OWNER OVER THEIR RESPECTIVE AREAS IN BORACAY, SINCE TIME Our Ruling
IMMEMORIAL OR AT THE LATEST SINCE 30 YRS. PRIOR TO THE FILING OF THE
PETITION FOR DECLARATORY RELIEF ON NOV. 19, 1997, WERE THE AREAS Regalian Doctrine and power of the executive
OCCUPIED BY THEM PUBLIC AGRICULTURAL LANDS AS DEFINED BY LAWS
THEN ON JUDICIAL CONFIRMATION OF IMPERFECT TITLES OR PUBLIC FOREST to reclassify lands of the public domain
AS DEFINED BY SEC. 3a, PD 705?
Private claimants rely on three (3) laws and executive acts in their bid for judicial
II. confirmation of imperfect title, namely: (a) Philippine Bill of 190236 in relation to Act No.
926, later amended and/or superseded by Act No. 2874 and CA No. 141; 37 (b)
Proclamation No. 180138 issued by then President Marcos; and (c) Proclamation No.
18 | P a g e
NAT RES & ENVI LAW CASES 2A_2023
106439 issued by President Gloria Macapagal-Arroyo. We shall proceed to determine their The Royal Decree of 1894 or the Maura Law53 partly amended the Spanish Mortgage Law
rights to apply for judicial confirmation of imperfect title under these laws and executive and the Laws of the Indies. It established possessory information as the method of
acts. legalizing possession of vacant Crown land, under certain conditions which were set forth
in said decree.54 Under Section 393 of the Maura Law, an informacion posesoria or
But first, a peek at the Regalian principle and the power of the executive to reclassify lands possessory information title,55 when duly inscribed in the Registry of Property, is converted
of the public domain. into a title of ownership only after the lapse of twenty (20) years of uninterrupted
possession which must be actual, public, and adverse, 56 from the date of its
The 1935 Constitution classified lands of the public domain into agricultural, forest or inscription.57 However, possessory information title had to be perfected one year after the
timber.40 Meanwhile, the 1973 Constitution provided the following divisions: agricultural, promulgation of the Maura Law, or until April 17, 1895. Otherwise, the lands would revert
industrial or commercial, residential, resettlement, mineral, timber or forest and grazing to the State.58
lands, and such other classes as may be provided by law, 41 giving the government great
leeway for classification.42 Then the 1987 Constitution reverted to the 1935 Constitution In sum, private ownership of land under the Spanish regime could only be founded on
classification with one addition: national parks.43 Of these, only agricultural lands may be royal concessions which took various forms, namely: (1) titulo real or royal grant;
alienated.44 Prior to Proclamation No. 1064 of May 22, 2006, Boracay Island (2) concesion especial or special grant; (3) composicion con el estado or adjustment title;
had never been expressly and administratively classified under any of these grand (4) titulo de compra or title by purchase; and (5) informacion posesoria or possessory
divisions. Boracay was an unclassified land of the public domain. information title.59>

The Regalian Doctrine dictates that all lands of the public domain belong to the State, that The first law governing the disposition of public lands in the Philippines under American
the State is the source of any asserted right to ownership of land and charged with the rule was embodied in the Philippine Bill of 1902.60 By this law, lands of the public domain
conservation of such patrimony.45 The doctrine has been consistently adopted under the in the Philippine Islands were classified into three (3) grand divisions, to wit: agricultural,
1935, 1973, and 1987 Constitutions.46 mineral, and timber or forest lands.61 The act provided for, among others, the disposal of
mineral lands by means of absolute grant (freehold system) and by lease (leasehold
All lands not otherwise appearing to be clearly within private ownership are presumed to system).62 It also provided the definition by exclusion of "agricultural public
belong to the State.47 Thus, all lands that have not been acquired from the government, lands."63 Interpreting the meaning of "agricultural lands" under the Philippine Bill of 1902,
either by purchase or by grant, belong to the State as part of the inalienable public the Court declared in Mapa v. Insular Government:64
domain.48 Necessarily, it is up to the State to determine if lands of the public domain will
be disposed of for private ownership. The government, as the agent of the state, is x x x In other words, that the phrase "agricultural land" as used in Act No. 926 means
possessed of the plenary power as the persona in law to determine who shall be the those public lands acquired from Spain which are not timber or mineral lands. x x
favored recipients of public lands, as well as under what terms they may be granted such x65 (Emphasis Ours)
privilege, not excluding the placing of obstacles in the way of their exercise of what
otherwise would be ordinary acts of ownership.49 On February 1, 1903, the Philippine Legislature passed Act No. 496, otherwise known as
the Land Registration Act. The act established a system of registration by which recorded
Our present land law traces its roots to the Regalian Doctrine. Upon the Spanish conquest title becomes absolute, indefeasible, and imprescriptible. This is known as the Torrens
of the Philippines, ownership of all lands, territories and possessions in the Philippines system.66
passed to the Spanish Crown.50 The Regalian doctrine was first introduced in the
Philippines through the Laws of the Indies and the Royal Cedulas, which laid the Concurrently, on October 7, 1903, the Philippine Commission passed Act No. 926, which
foundation that "all lands that were not acquired from the Government, either by purchase was the first Public Land Act. The Act introduced the homestead system and made
or by grant, belong to the public domain."51 provisions for judicial and administrative confirmation of imperfect titles and for the sale or
lease of public lands. It permitted corporations regardless of the nationality of persons
The Laws of the Indies was followed by the Ley Hipotecaria or the Mortgage Law of owning the controlling stock to lease or purchase lands of the public domain. 67 Under the
1893. The Spanish Mortgage Law provided for the systematic registration of titles and Act, open, continuous, exclusive, and notorious possession and occupation of agricultural
deeds as well as possessory claims.52 lands for the next ten (10) years preceding July 26, 1904 was sufficient for judicial
confirmation of imperfect title.68
19 | P a g e
NAT RES & ENVI LAW CASES 2A_2023
On November 29, 1919, Act No. 926 was superseded by Act No. 2874, otherwise known declaring land of the public domain as alienable and disposable. To prove that the land
as the second Public Land Act. This new, more comprehensive law limited the exploitation subject of an application for registration is alienable, the applicant must establish the
of agricultural lands to Filipinos and Americans and citizens of other countries which gave existence of a positive act of the government such as a presidential proclamation or an
Filipinos the same privileges. For judicial confirmation of title, possession and executive order; an administrative action; investigation reports of Bureau of Lands
occupation en concepto dueño since time immemorial, or since July 26, 1894, was investigators; and a legislative act or a statute.85 The applicant may also secure a
required.69 certification from the government that the land claimed to have been possessed for the
required number of years is alienable and disposable.86
After the passage of the 1935 Constitution, CA No. 141 amended Act No. 2874
on December 1, 1936. To this day, CA No. 141, as amended, remains as the existing In the case at bar, no such proclamation, executive order, administrative action, report,
general law governing the classification and disposition of lands of the public domain other statute, or certification was presented to the Court. The records are bereft of evidence
than timber and mineral lands,70 and privately owned lands which reverted to the State.71 showing that, prior to 2006, the portions of Boracay occupied by private claimants were
subject of a government proclamation that the land is alienable and disposable. Absent
Section 48(b) of CA No. 141 retained the requirement under Act No. 2874 of possession such well-nigh incontrovertible evidence, the Court cannot accept the submission that
and occupation of lands of the public domain since time immemorial or since July 26, 1894. lands occupied by private claimants were already open to disposition before 2006. Matters
However, this provision was superseded by Republic Act (RA) No. 1942,72 which provided of land classification or reclassification cannot be assumed. They call for proof.87
for a simple thirty-year prescriptive period for judicial confirmation of imperfect title. The
provision was last amended by PD No. 1073,73 which now provides for possession and Ankron and De Aldecoa did not make the whole of Boracay Island, or portions of it,
occupation of the land applied for since June 12, 1945, or earlier.74 agricultural lands. Private claimants posit that Boracay was already an agricultural land
pursuant to the old cases Ankron v. Government of the Philippine Islands (1919)88 and De
The issuance of PD No. 89275 on February 16, 1976 discontinued the use of Spanish titles Aldecoa v. The Insular Government (1909).89 These cases were decided under the
as evidence in land registration proceedings.76 Under the decree, all holders of Spanish provisions of the Philippine Bill of 1902 and Act No. 926. There is a statement in these old
titles or grants should apply for registration of their lands under Act No. 496 within six (6) cases that "in the absence of evidence to the contrary, that in each case the lands are
months from the effectivity of the decree on February 16, 1976. Thereafter, the recording agricultural lands until the contrary is shown."90
of all unregistered lands77 shall be governed by Section 194 of the Revised Administrative
Code, as amended by Act No. 3344. Private claimants’ reliance on Ankron and De Aldecoa is misplaced. These cases did not
have the effect of converting the whole of Boracay Island or portions of it into agricultural
On June 11, 1978, Act No. 496 was amended and updated by PD No. 1529, known as the lands. It should be stressed that the Philippine Bill of 1902 and Act No. 926 merely provided
Property Registration Decree. It was enacted to codify the various laws relative to the manner through which land registration courts would classify lands of the public domain.
registration of property.78 It governs registration of lands under the Torrens system as well Whether the land would be classified as timber, mineral, or agricultural depended on proof
as unregistered lands, including chattel mortgages.79 presented in each case.

A positive act declaring land as alienable and disposable is required. In keeping with Ankron and De Aldecoa were decided at a time when the President of the Philippines had
the presumption of State ownership, the Court has time and again emphasized that there no power to classify lands of the public domain into mineral, timber, and agricultural. At
must be a positive act of the government, such as an official that time, the courts were free to make corresponding classifications in justiciable cases,
proclamation,80 declassifying inalienable public land into disposable land for agricultural or or were vested with implicit power to do so, depending upon the preponderance of the
other purposes.81 In fact, Section 8 of CA No. 141 limits alienable or disposable lands only evidence.91 This was the Court’s ruling in Heirs of the Late Spouses Pedro S. Palanca and
to those lands which have been "officially delimited and classified."82 Soterranea Rafols Vda. De Palanca v. Republic,92 in which it stated, through Justice Adolfo
Azcuna, viz.:
The burden of proof in overcoming the presumption of State ownership of the lands of the
public domain is on the person applying for registration (or claiming ownership), who must x x x Petitioners furthermore insist that a particular land need not be formally released by
prove that the land subject of the application is alienable or disposable.83 To overcome this an act of the Executive before it can be deemed open to private ownership, citing the cases
presumption, incontrovertible evidence must be established that the land subject of the of Ramos v. Director of Lands and Ankron v. Government of the Philippine Islands.
application (or claim) is alienable or disposable.84 There must still be a positive act
20 | P a g e
NAT RES & ENVI LAW CASES 2A_2023
xxxx question of fact. The mere fact that a tract of land has trees upon it or has mineral within it
is not of itself sufficient to declare that one is forestry land and the other, mineral land.
Petitioner’s reliance upon Ramos v. Director of Lands and Ankron v. Government is There must be some proof of the extent and present or future value of the forestry and of
misplaced. These cases were decided under the Philippine Bill of 1902 and the first Public the minerals. While, as we have just said, many definitions have been given for
Land Act No. 926 enacted by the Philippine Commission on October 7, 1926, under which "agriculture," "forestry," and "mineral" lands, and that in each case it is a question of fact,
there was no legal provision vesting in the Chief Executive or President of the Philippines we think it is safe to say that in order to be forestry or mineral land the proof must show
the power to classify lands of the public domain into mineral, timber and agricultural so that it is more valuable for the forestry or the mineral which it contains than it is for
that the courts then were free to make corresponding classifications in justiciable cases, agricultural purposes. (Sec. 7, Act No. 1148.) It is not sufficient to show that there exists
or were vested with implicit power to do so, depending upon the preponderance of the some trees upon the land or that it bears some mineral. Land may be classified as forestry
evidence.93 or mineral today, and, by reason of the exhaustion of the timber or mineral, be classified
as agricultural land tomorrow. And vice-versa, by reason of the rapid growth of timber or
To aid the courts in resolving land registration cases under Act No. 926, it was then the discovery of valuable minerals, lands classified as agricultural today may be differently
necessary to devise a presumption on land classification. Thus evolved the dictum classified tomorrow. Each case must be decided upon the proof in that particular case,
in Ankron that "the courts have a right to presume, in the absence of evidence to the having regard for its present or future value for one or the other purposes. We
contrary, that in each case the lands are agricultural lands until the contrary is shown."94 believe, however, considering the fact that it is a matter of public knowledge that a majority
of the lands in the Philippine Islands are agricultural lands that the courts have a right to
presume, in the absence of evidence to the contrary, that in each case the lands are
But We cannot unduly expand the presumption in Ankron and De Aldecoa to an argument
agricultural lands until the contrary is shown. Whatever the land involved in a particular
that all lands of the public domain had been automatically reclassified as disposable and
land registration case is forestry or mineral land must, therefore, be a matter of proof.
alienable agricultural lands. By no stretch of imagination did the presumption convert all
Its superior value for one purpose or the other is a question of fact to be settled by
lands of the public domain into agricultural lands.
the proof in each particular case. The fact that the land is a manglar [mangrove swamp]
is not sufficient for the courts to decide whether it is agricultural, forestry, or mineral land.
If We accept the position of private claimants, the Philippine Bill of 1902 and Act No. 926 It may perchance belong to one or the other of said classes of land. The Government, in
would have automatically made all lands in the Philippines, except those already classified the first instance, under the provisions of Act No. 1148, may, by reservation, decide for
as timber or mineral land, alienable and disposable lands. That would take these lands out itself what portions of public land shall be considered forestry land, unless private interests
of State ownership and worse, would be utterly inconsistent with and totally repugnant to have intervened before such reservation is made. In the latter case, whether the land is
the long-entrenched Regalian doctrine. agricultural, forestry, or mineral, is a question of proof. Until private interests have
intervened, the Government, by virtue of the terms of said Act (No. 1148), may decide for
The presumption in Ankron and De Aldecoa attaches only to land registration cases itself what portions of the "public domain" shall be set aside and reserved as forestry or
brought under the provisions of Act No. 926, or more specifically those cases dealing with mineral land. (Ramos vs. Director of Lands, 39 Phil. 175; Jocson vs. Director of
judicial and administrative confirmation of imperfect titles. The presumption applies to an Forestry, supra)95 (Emphasis ours)
applicant for judicial or administrative conformation of imperfect title under Act No. 926. It
certainly cannot apply to landowners, such as private claimants or their predecessors-in- Since 1919, courts were no longer free to determine the classification of lands from the
interest, who failed to avail themselves of the benefits of Act No. 926. As to them, their facts of each case, except those that have already became private lands.96 Act
land remained unclassified and, by virtue of the Regalian doctrine, continued to be owned No. 2874, promulgated in 1919 and reproduced in Section 6 of CA No. 141, gave the
by the State. Executive Department, through the President, the exclusive prerogative to classify or
reclassify public lands into alienable or disposable, mineral or forest.96-a Since then, courts
In any case, the assumption in Ankron and De Aldecoa was not absolute. Land no longer had the authority, whether express or implied, to determine the classification of
classification was, in the end, dependent on proof. If there was proof that the land was lands of the public domain.97
better suited for non-agricultural uses, the courts could adjudge it as a mineral or timber
land despite the presumption. In Ankron, this Court stated: Here, private claimants, unlike the Heirs of Ciriaco Tirol who were issued their title in
1933,98 did not present a justiciable case for determination by the land registration court
In the case of Jocson vs. Director of Forestry (supra), the Attorney-General admitted in of the property’s land classification. Simply put, there was no opportunity for the courts
effect that whether the particular land in question belongs to one class or another is a
21 | P a g e
NAT RES & ENVI LAW CASES 2A_2023
then to resolve if the land the Boracay occupants are now claiming were agricultural lands. cancellation or confirmation of Spanish concessions and grants in the Islands." In short,
When Act No. 926 was supplanted by Act No. 2874 in 1919, without an application for the Public Land Act operated on the assumption that title to public lands in the Philippine
judicial confirmation having been filed by private claimants or their predecessors-in- Islands remained in the government; and that the government’s title to public land sprung
interest, the courts were no longer authorized to determine the property’s land from the Treaty of Paris and other subsequent treaties between Spain and the United
classification. Hence, private claimants cannot bank on Act No. 926. States. The term "public land" referred to all lands of the public domain whose title still
remained in the government and are thrown open to private appropriation and settlement,
We note that the RTC decision99 in G.R. No. 167707 mentioned Krivenko v. Register of and excluded the patrimonial property of the government and the friar lands."
Deeds of Manila,100 which was decided in 1947 when CA No. 141, vesting the Executive
with the sole power to classify lands of the public domain was already in Thus, it is plain error for petitioners to argue that under the Philippine Bill of 1902
effect. Krivenko cited the old cases Mapa v. Insular Government,101 De Aldecoa v. The and Public Land Act No. 926, mere possession by private individuals of lands
Insular Government,102 and Ankron v. Government of the Philippine Islands.103 creates the legal presumption that the lands are alienable and
disposable.108 (Emphasis Ours)
Krivenko, however, is not controlling here because it involved a totally different issue. The
pertinent issue in Krivenko was whether residential lots were included in the general Except for lands already covered by existing titles, Boracay was an unclassified
classification of agricultural lands; and if so, whether an alien could acquire a residential land of the public domain prior to Proclamation No. 1064. Such unclassified lands
lot. This Court ruled that as an alien, Krivenko was prohibited by the 1935 are considered public forest under PD No. 705. The DENR109 and the National Mapping
Constitution104 from acquiring agricultural land, which included residential lots. Here, the and Resource Information Authority110 certify that Boracay Island is an unclassified land of
issue is whether unclassified lands of the public domain are automatically deemed the public domain.
agricultural.
PD No. 705 issued by President Marcos categorized all unclassified lands of the public
Notably, the definition of "agricultural public lands" mentioned in Krivenko relied on the old domain as public forest. Section 3(a) of PD No. 705 defines a public forest as "a mass of
cases decided prior to the enactment of Act No. 2874, including Ankron and De lands of the public domain which has not been the subject of the present system of
Aldecoa.105 As We have already stated, those cases cannot apply here, since they were classification for the determination of which lands are needed for forest purpose and which
decided when the Executive did not have the authority to classify lands as agricultural, are not." Applying PD No. 705, all unclassified lands, including those in Boracay Island,
timber, or mineral. are ipso facto considered public forests. PD No. 705, however, respects titles already
existing prior to its effectivity.
Private claimants’ continued possession under Act No. 926 does not create a
presumption that the land is alienable. Private claimants also contend that their The Court notes that the classification of Boracay as a forest land under PD No. 705 may
continued possession of portions of Boracay Island for the requisite period of ten (10) years seem to be out of touch with the present realities in the island. Boracay, no doubt, has
under Act No. 926106 ipso facto converted the island into private ownership. Hence, they been partly stripped of its forest cover to pave the way for commercial developments. As
may apply for a title in their name. a premier tourist destination for local and foreign tourists, Boracay appears more of a
commercial island resort, rather than a forest land.
A similar argument was squarely rejected by the Court in Collado v. Court of
Appeals.107 Collado, citing the separate opinion of now Chief Justice Reynato S. Puno Nevertheless, that the occupants of Boracay have built multi-million peso beach resorts on
in Cruz v. Secretary of Environment and Natural Resources,107-a ruled: the island;111 that the island has already been stripped of its forest cover; or that the
implementation of Proclamation No. 1064 will destroy the island’s tourism industry,
"Act No. 926, the first Public Land Act, was passed in pursuance of the provisions of the do not negate its character as public forest.
Philippine Bill of 1902. The law governed the disposition of lands of the public domain. It
prescribed rules and regulations for the homesteading, selling and leasing of portions of Forests, in the context of both the Public Land Act and the Constitution112 classifying lands
the public domain of the Philippine Islands, and prescribed the terms and conditions to of the public domain into "agricultural, forest or timber, mineral lands, and national parks,"
enable persons to perfect their titles to public lands in the Islands. It also provided for the do not necessarily refer to large tracts of wooded land or expanses covered by dense
"issuance of patents to certain native settlers upon public lands," for the establishment of growths of trees and underbrushes.113 The discussion in Heirs of Amunategui v. Director
town sites and sale of lots therein, for the completion of imperfect titles, and for the of Forestry114 is particularly instructive:
22 | P a g e
NAT RES & ENVI LAW CASES 2A_2023
A forested area classified as forest land of the public domain does not lose such Subsistence farming, in areas declared as alienable and disposable by the Bureau of
classification simply because loggers or settlers may have stripped it of its forest cover. Forest Development.
Parcels of land classified as forest land may actually be covered with grass or planted to
crops by kaingin cultivators or other farmers. "Forest lands" do not have to be on mountains Therefore, Proclamation No. 1801 cannot be deemed the positive act needed to classify
or in out of the way places. Swampy areas covered by mangrove trees, nipa palms, and Boracay Island as alienable and disposable land. If President Marcos intended to classify
other trees growing in brackish or sea water may also be classified as forest land. The the island as alienable and disposable or forest, or both, he would have identified the
classification is descriptive of its legal nature or status and does not have to be specific limits of each, as President Arroyo did in Proclamation No. 1064. This was not
descriptive of what the land actually looks like. Unless and until the land classified as done in Proclamation No. 1801.
"forest" is released in an official proclamation to that effect so that it may form part of the
disposable agricultural lands of the public domain, the rules on confirmation of imperfect The Whereas clauses of Proclamation No. 1801 also explain the rationale behind the
title do not apply.115 (Emphasis supplied) declaration of Boracay Island, together with other islands, caves and peninsulas in the
Philippines, as a tourist zone and marine reserve to be administered by the PTA – to
There is a big difference between "forest" as defined in a dictionary and "forest or timber ensure the concentrated efforts of the public and private sectors in the development of the
land" as a classification of lands of the public domain as appearing in our statutes. One is areas’ tourism potential with due regard for ecological balance in the marine environment.
descriptive of what appears on the land while the other is a legal status, a classification for Simply put, the proclamation is aimed at administering the islands for tourism and
legal purposes.116 At any rate, the Court is tasked to determine the legal status of Boracay ecological purposes. It does not address the areas’ alienability.119
Island, and not look into its physical layout. Hence, even if its forest cover has been
replaced by beach resorts, restaurants and other commercial establishments, it has not More importantly, Proclamation No. 1801 covers not only Boracay Island, but sixty-four
been automatically converted from public forest to alienable agricultural land. (64) other islands, coves, and peninsulas in the Philippines, such as Fortune and Verde
Islands in Batangas, Port Galera in Oriental Mindoro, Panglao and Balicasag Islands in
Private claimants cannot rely on Proclamation No. 1801 as basis for judicial Bohol, Coron Island, Puerto Princesa and surrounding areas in Palawan, Camiguin Island
confirmation of imperfect title. The proclamation did not convert Boracay into an in Cagayan de Oro, and Misamis Oriental, to name a few. If the designation of Boracay
agricultural land. However, private claimants argue that Proclamation No. 1801 issued Island as tourist zone makes it alienable and disposable by virtue of Proclamation No.
by then President Marcos in 1978 entitles them to judicial confirmation of imperfect title. 1801, all the other areas mentioned would likewise be declared wide open for private
The Proclamation classified Boracay, among other islands, as a tourist zone. Private disposition. That could not have been, and is clearly beyond, the intent of the proclamation.
claimants assert that, as a tourist spot, the island is susceptible of private ownership.
It was Proclamation No. 1064 of 2006 which positively declared part of Boracay as
Proclamation No. 1801 or PTA Circular No. 3-82 did not convert the whole of Boracay into alienable and opened the same to private ownership. Sections 6 and 7 of CA No.
an agricultural land. There is nothing in the law or the Circular which made Boracay Island 141120 provide that it is only the President, upon the recommendation of the proper
an agricultural land. The reference in Circular No. 3-82 to "private lands"117 and "areas department head, who has the authority to classify the lands of the public domain into
declared as alienable and disposable"118 does not by itself classify the entire island as alienable or disposable, timber and mineral lands.121
agricultural. Notably, Circular No. 3-82 makes reference not only to private lands and areas
but also to public forested lands. Rule VIII, Section 3 provides: In issuing Proclamation No. 1064, President Gloria Macapagal-Arroyo merely exercised
the authority granted to her to classify lands of the public domain, presumably subject to
No trees in forested private lands may be cut without prior authority from the PTA. All existing vested rights. Classification of public lands is the exclusive prerogative of the
forested areas in public lands are declared forest reserves. (Emphasis supplied) Executive Department, through the Office of the President. Courts have no authority to do
so.122 Absent such classification, the land remains unclassified until released and rendered
Clearly, the reference in the Circular to both private and public lands merely recognizes open to disposition.123
that the island can be classified by the Executive department pursuant to its powers under
CA No. 141. In fact, Section 5 of the Circular recognizes the then Bureau of Forest Proclamation No. 1064 classifies Boracay into 400 hectares of reserved forest land and
Development’s authority to declare areas in the island as alienable and disposable when 628.96 hectares of agricultural land. The Proclamation likewise provides for a 15-meter
it provides: buffer zone on each side of the center line of roads and trails, which are reserved for right
of way and which shall form part of the area reserved for forest land protection purposes.
23 | P a g e
NAT RES & ENVI LAW CASES 2A_2023
Contrary to private claimants’ argument, there was nothing invalid or irregular, much less prohibited reclassification under the agrarian law. We agree with the opinion of the
unconstitutional, about the classification of Boracay Island made by the President through Department of Justice126 on this point:
Proclamation No. 1064. It was within her authority to make such classification, subject to
existing vested rights. Indeed, the key word to the correct application of the prohibition in Section 4(a) is the word
"reclassification." Where there has been no previous classification of public forest
Proclamation No. 1064 does not violate the Comprehensive Agrarian Reform [referring, we repeat, to the mass of the public domain which has not been the subject of
Law. Private claimants further assert that Proclamation No. 1064 violates the provision of the present system of classification for purposes of determining which are needed for
the Comprehensive Agrarian Reform Law (CARL) or RA No. 6657 barring conversion of forest purposes and which are not] into permanent forest or forest reserves or some other
public forests into agricultural lands. They claim that since Boracay is a public forest under forest uses under the Revised Forestry Code, there can be no "reclassification of forest
PD No. 705, President Arroyo can no longer convert it into an agricultural land without lands" to speak of within the meaning of Section 4(a).
running afoul of Section 4(a) of RA No. 6657, thus:
Thus, obviously, the prohibition in Section 4(a) of the CARL against the reclassification of
SEC. 4. Scope. – The Comprehensive Agrarian Reform Law of 1988 shall cover, forest lands to agricultural lands without a prior law delimiting the limits of the public domain,
regardless of tenurial arrangement and commodity produced, all public and private does not, and cannot, apply to those lands of the public domain, denominated as "public
agricultural lands as provided in Proclamation No. 131 and Executive Order No. 229, forest" under the Revised Forestry Code, which have not been previously determined, or
including other lands of the public domain suitable for agriculture. classified, as needed for forest purposes in accordance with the provisions of the Revised
Forestry Code.127
More specifically, the following lands are covered by the Comprehensive Agrarian Reform
Program: Private claimants are not entitled to apply for judicial confirmation of imperfect title
under CA No. 141. Neither do they have vested rights over the occupied lands under
(a) All alienable and disposable lands of the public domain devoted to or suitable for the said law. There are two requisites for judicial confirmation of imperfect or incomplete
agriculture. No reclassification of forest or mineral lands to agricultural lands shall be title under CA No. 141, namely: (1) open, continuous, exclusive, and notorious possession
undertaken after the approval of this Act until Congress, taking into account ecological, and occupation of the subject land by himself or through his predecessors-in-interest under
developmental and equity considerations, shall have determined by law, the specific limits a bona fide claim of ownership since time immemorial or from June 12, 1945; and (2) the
of the public domain. classification of the land as alienable and disposable land of the public domain.128

That Boracay Island was classified as a public forest under PD No. 705 did not bar the As discussed, the Philippine Bill of 1902, Act No. 926, and Proclamation No. 1801 did not
Executive from later converting it into agricultural land. Boracay Island still remained an convert portions of Boracay Island into an agricultural land. The island remained an
unclassified land of the public domain despite PD No. 705. unclassified land of the public domain and, applying the Regalian doctrine, is considered
State property.
In Heirs of the Late Spouses Pedro S. Palanca and Soterranea Rafols v. Republic, 124 the
Court stated that unclassified lands are public forests. Private claimants’ bid for judicial confirmation of imperfect title, relying on the Philippine
Bill of 1902, Act No. 926, and Proclamation No. 1801, must fail because of the absence of
While it is true that the land classification map does not categorically state that the the second element of alienable and disposable land. Their entitlement to a government
islands are public forests, the fact that they were unclassified lands leads to the grant under our present Public Land Act presupposes that the land possessed and applied
same result. In the absence of the classification as mineral or timber land, the land for is already alienable and disposable. This is clear from the wording of the law
remains unclassified land until released and rendered open to disposition.125 (Emphasis itself.129 Where the land is not alienable and disposable, possession of the land, no matter
supplied) how long, cannot confer ownership or possessory rights.130

Moreover, the prohibition under the CARL applies only to a "reclassification" of land. If the Neither may private claimants apply for judicial confirmation of imperfect title under
land had never been previously classified, as in the case of Boracay, there can be no Proclamation No. 1064, with respect to those lands which were classified as agricultural
lands. Private claimants failed to prove the first element of open, continuous, exclusive,
and notorious possession of their lands in Boracay since June 12, 1945.
24 | P a g e
NAT RES & ENVI LAW CASES 2A_2023
We cannot sustain the CA and RTC conclusion in the petition for declaratory relief that More realistically, Congress may enact a law to entitle private claimants to acquire title to
private claimants complied with the requisite period of possession. their occupied lots or to exempt them from certain requirements under the present land
laws. There is one such bill133 now pending in the House of Representatives. Whether that
The tax declarations in the name of private claimants are insufficient to prove the first bill or a similar bill will become a law is for Congress to decide.
element of possession. We note that the earliest of the tax declarations in the name of
private claimants were issued in 1993. Being of recent dates, the tax declarations are not In issuing Proclamation No. 1064, the government has taken the step necessary to open
sufficient to convince this Court that the period of possession and occupation commenced up the island to private ownership. This gesture may not be sufficient to appease some
on June 12, 1945. sectors which view the classification of the island partially into a forest reserve as absurd.
That the island is no longer overrun by trees, however, does not becloud the vision to
Private claimants insist that they have a vested right in Boracay, having been in possession protect its remaining forest cover and to strike a healthy balance between progress and
of the island for a long time. They have invested millions of pesos in developing the island ecology. Ecological conservation is as important as economic progress.
into a tourist spot. They say their continued possession and investments give them a
vested right which cannot be unilaterally rescinded by Proclamation No. 1064. To be sure, forest lands are fundamental to our nation’s survival. Their promotion and
protection are not just fancy rhetoric for politicians and activists. These are needs that
The continued possession and considerable investment of private claimants do not become more urgent as destruction of our environment gets prevalent and difficult to
automatically give them a vested right in Boracay. Nor do these give them a right to apply control. As aptly observed by Justice Conrado Sanchez in 1968 in Director of Forestry v.
for a title to the land they are presently occupying. This Court is constitutionally bound to Munoz:134
decide cases based on the evidence presented and the laws applicable. As the law and
jurisprudence stand, private claimants are ineligible to apply for a judicial confirmation of The view this Court takes of the cases at bar is but in adherence to public policy that should
title over their occupied portions in Boracay even with their continued possession and be followed with respect to forest lands. Many have written much, and many more have
considerable investment in the island. spoken, and quite often, about the pressing need for forest preservation, conservation,
protection, development and reforestation. Not without justification. For, forests constitute
One Last Note a vital segment of any country's natural resources. It is of common knowledge by now that
absence of the necessary green cover on our lands produces a number of adverse or ill
The Court is aware that millions of pesos have been invested for the development of effects of serious proportions. Without the trees, watersheds dry up; rivers and lakes which
Boracay Island, making it a by-word in the local and international tourism industry. The they supply are emptied of their contents. The fish disappear. Denuded areas become dust
Court also notes that for a number of years, thousands of people have called the island bowls. As waterfalls cease to function, so will hydroelectric plants. With the rains, the fertile
their home. While the Court commiserates with private claimants’ plight, We are bound to topsoil is washed away; geological erosion results. With erosion come the dreaded floods
apply the law strictly and judiciously. This is the law and it should prevail. Ito ang batas at that wreak havoc and destruction to property – crops, livestock, houses, and highways –
ito ang dapat umiral. not to mention precious human lives. Indeed, the foregoing observations should be written
down in a lumberman’s decalogue.135
All is not lost, however, for private claimants. While they may not be eligible to apply for
judicial confirmation of imperfect title under Section 48(b) of CA No. 141, as amended, this WHEREFORE, judgment is rendered as follows:
does not denote their automatic ouster from the residential, commercial, and other areas
they possess now classified as agricultural. Neither will this mean the loss of their 1. The petition for certiorari in G.R. No. 167707 is GRANTED and the Court of Appeals
substantial investments on their occupied alienable lands. Lack of title does not necessarily Decision in CA-G.R. CV No. 71118 REVERSED AND SET ASIDE.
mean lack of right to possess.
2. The petition for certiorari in G.R. No. 173775 is DISMISSED for lack of merit.
For one thing, those with lawful possession may claim good faith as builders of
improvements. They can take steps to preserve or protect their possession. For another, SO ORDERED.
they may look into other modes of applying for original registration of title, such as by
homestead131 or sales patent,132 subject to the conditions imposed by law.

25 | P a g e
NAT RES & ENVI LAW CASES 2A_2023
5.
Thereafter, a 2,445-square-meter portion of Lot 1 (Lot 1-A[11] was transferred to
G.R. No. 218418. November 08, 2017 Consolacion.[12] Thus, on April 6, 1982, Transfer Certificate of Title (TCT) No. 16580
REPUBLIC OF THE PHILIPPINES, REPRESENTED BY THE REGIONAL EXECUTIVE covering Lot 1-A was issued in Consolacion's name.[13] Later still, Consolacion sold
DIRECTOR, DENR, REGION IV, MANILA, PETITIONER, portions of Lot 1-A to several purchasers namely: Dy, Agbayani, Soriano, Calma, and
V. Liwanag.[14]
THE HEIRS OF MEYNARDO CABRERA, AS HEREIN REPRESENTED BY
MEYNARDO CABRERA, JR. AND ALMA RODRIGUEZ CABRERA, THE HEIRS OF Learning of the issuance of TCT No. 16580, Jose and Leticia De Castro (De Castros),
CONSOLACION DIMACULANGAN CABRERA, AS HEREIN REPRESENTED BY claiming to be the actual possessors of Lot 1-A, filed before the Department of
ALEXANDER CABRERA, MANIBI CABRERA, MILAGROS CABRERA GARA, AND Environment and Natural Resources (DENR) a petition urging DENR to conduct an
RAUL CABRERA, JACKSON CINCO DY, LORETA AGBAYANI, GLORIA SORIANO, investigation to determine Lot 1-A's land classification status.[15]
CRIS CALMA, NORA LIWANAG AND THE REGISTER OF DEEDS OF ORIENTAL
MINDORO, RESPONDENTS. Consequently, in the DENR Final Investigation Report[16] (DENR Final Report) dated
November 9, 1994 issued by Erwin D. Talento of the DENR Land Management Office
DECISION (LMO), Free Patent No. 516197, covering Lots 1, 1-A, and 2 (collectively, Roxas
CAGUIOA, J: Properties), was declared null and void for having been issued over land forming part of
the public domain. The pertinent portions of the DENR Final Report read:
The Case
This is a Petition for Review on Certiorari[1] (Petition) filed under Rule 45 of the Rules of Sensing that they don't have any chance in the court to prove their better right to occupy
Court against the Decision[2] dated July 18, 2014 (Assailed Decision) and Resolution[3] and possess [Lot 1-A] x x x the [De Castros] addressed their petitions to the DENR basing
dated May 20, 2015 (Assailed Resolution) in CA-G.R. CV No. 98120 rendered by the Court their claim on the weight of a certification of [the National Mapping and Resource
of Appeals (CA) Eleventh Division and Special Former Eleventh Division, respectively. Information Authority (NAMRIA)] x x x. The [De Castros] are now seeking administrative
remedies for the issue which they have already brought to the attention of the court and
The Assailed Decision and Resolution stem from an appeal from the Decision[4] dated wherein they have failed to prove their priority right to occupy and possess [Lot 1-A].
December 5, 2005 rendered by the Regional Trial Court of Roxas, Oriental Mindoro, Granting that [the Roxas Properties constitute] forest land and [Free Patent No. 516197]
Branch 43 (RTC) in Civil Case No. C-358, dismissing the complaint for cancellation of free issued in favor of [Meynardo] be (sic) rendered null and void [ab] initio, it (sic) doesn't
patent and reversion filed by the Republic of the Philippines (Republic) against the Heirs warrant that they have better right to possess and occupy [Lot 1-A] because [Meynardo,
of Meynardo Cabrera (Heirs of Meynardo), the Heirs of Consolacion Dimaculangan through his predecessors-in-interest] have entered [Lot 1-A] since the year 1943 and have
Cabrera (Heirs of Consolacion), Jackson Cinco Dy (Dy), Loreta Agbayani (Agbayani), exercised their ownership over the same x x x.
Gloria Soriano (Soriano), Cris Calma (Calma), Nora Liwanag (Liwanag), and the Register
of Deeds of Oriental Mindoro (ROD) (collectively, Respondents).[5] In view of the foregoing, it is respectfully recommended that the petition of [the De Castros]
be dismissed x x x and appropriate legal action be instituted for the cancellation of Free
The Facts Patent No. 516197 issued in favor of Meynardo x x x for the same covers land of the public
domain which is certified by the proper authority as public forest.[17] (Emphasis supplied.)
Sometime in 1971, Meynardo filed an Application for Free Patent concerning an 8,072[6]
square-meter parcel of land situated in Pining, Roxas, Oriental Mindoro.[7] In said Thereafter, Antonio G. Principe, the DENR Regional Executive Director of Region IV,
application, Meynardo alleged that he had been in possession of such parcel of land since issued an Order[18] dated August 8, 1997 declaring Free Patent No. 516197 null and void.
1936, through his predecessor-in- interest Marcelo Cabrera.[8]
Later, on November 15, 1999, the Republic filed against the Respondents a complaint
In the same year, the Bureau of Lands (BOL) issued Free Patent No. 516197 in favor of (Complaint) for the annulment and/or cancellation of Free Patent No. 516197, OCT No.
Meynardo, covering two (2) lots denominated as: (i) Lot 1 with an area of 3,591 square RP-132 (P-9193), and TCT No. 16580. The Complaint also prayed for the reversion of the
meters, and (ii) Lot 2, with an area of 4,481 square meters.[9] On the basis of said patent, Roxas Properties in the State's favor.[19]
the ROD issued Original Certificate of Title (OCT) No. RP-132 (P-9193) covering both lots
in Meynardo's name.[10]
26 | P a g e
NAT RES & ENVI LAW CASES 2A_2023
The Republic based its claim on the (i) DENR Final Report; and (ii) NAMRIA certifications classification of forest land to alienable and disposable land, and not the other way around,
dated January 31, 1994, February 1, 1994, and October 3, 1994, all stating that the Roxas as in this case.[27] Further, the Republic averred that fraud must have necessarily
Properties (including Lot 1-A) had been reclassified as forest land as early as November attended the issuance of Free Patent No. 516197, OCT No. RP-132 and TCT No. 16580,
24, 1949. The statements in these documents were, in turn, based on the inscriptions owing to the status of the Roxas Properties as forest land.[28]
appearing on Land Classification Map No. 209 (LC Map 209) dated March 6, 1924
covering the Roxas Properties. The Republic reasoned that while LC Map 209 indicates On July 18, 2014, the CA rendered the Assailed Decision dismissing the Appeal. The
that the parcels of land thereunder were classified as alienable and disposable at the time dispositive portion of said decision reads:
it was prepared, a subsequent annotation made thereon indicates that they were
reclassified as forest land sometime thereafter, and had thus become inalienable.[20] WHEREFORE, premises considered, the Appeal is DISMISSED. The Decision dated
December 5, 2005 of the [RTC] x x x is AFFIRMED.
In their respective answers, the Respondents averred, among others, that: (i) Lot 1-A forms
part of the alienable and disposable land of the public domain, as evidenced by the original SO ORDERED.[29]
statements appearing on LC Map 209; (ii) the annotations appearing on LC Map 209 do
not serve as sufficient proof of reversion; and (iii) the land area which had been purportedly According to the CA, the Public Land Act vests the power to classify (and reclassify) lands
reclassified as forest land was not properly identified since the Republic failed to present of the public domain with the President. On this score, the CA held that the annotations
the technical description corresponding thereto.[21] In addition to these common appearing on LC Map 209 anent the alleged reversion of the Roxas Properties deserve
assertions, respondents Dy, Agbayani, Soriano, and Liwanag further averred that they scant consideration, as they do not appear to be based on any executive directive.
acquired portions of Lot 1-A from Consolacion in good faith, and have, since then, been in Consequently, the NAMRIA certifications and DENR Final Report relied upon by the
actual, exclusive, open, and continuous possession of their respective portions as Republic are insufficient to sustain its cause, as they are, in turn, based solely on said
owners.[22] annotations.[30]

On December 5, 2005, the RTC rendered a Decision, the dispositive portion of which The Republic filed an MR, which was denied by the CA in its Assailed Resolution dated
states: May 20, 2015. The Republic received a copy of the Assailed Resolution on June 8,
2015.[31]
ACCORDINGLY, judgment is hereby rendered DISMISSING the instant complaint for lack
of merit. On June 19, 2015, the Republic filed a Motion for Extension of Time to File Petition for
Review, praying for an additional period of twenty-five (25) days from June 23, 2015, or
SO ORDERED.[23] until July 18, 2015 within which to file a petition for review on certiorari. Subsequently, the
Republic filed a Second Motion for Extension, praying for a five (5)-day extension.[32]
The RTC found that the Republic failed to present proof that the Roxas Properties
(including Lot 1-A) have been reclassified as forest land. Citing Republic v. Animas,[24] Finally, on July 22, 2015, the Republic filed the present Petition, to which Respondents
(Animas) the RTC held that in order to prove reversion of alienable and disposable land to filed their Compliance and Comment dated December 16, 2016.[33]
forest land, a positive government act evincing the same is necessary.[25]
Thereafter, the Republic filed a Manifestation and Motion dated May 28, 2017, adopting
The Republic filed a motion for reconsideration (MR), which was denied in the RTC's Order the Petition as its reply to Respondents' Compliance and Comment.[34]
dated October 18, 2011.[26]
The Issue
CA Proceedings
The Petition calls on the Court to determine whether the CA erred when it held that a
Aggrieved, the Republic elevated the case to the CA via petition for review under Rule 42, positive act of government is necessary to evince the reclassification of land from alienable
docketed as CA-G.R. CV No. 98120 (Appeal). and disposable to forest.

In the Appeal, the Republic argued that the Court's ruling in Animas cannot be applied to The Court's Ruling
the present case, since, in the former, the fact sought to be established was the
27 | P a g e
NAT RES & ENVI LAW CASES 2A_2023
In this Petition, the Republic maintains that the Court's ruling in Animas did not have the of title may be recovered by the State through reversion proceedings, in accordance with
effect of making a positive executive act a necessary requirement for the purpose of the Public Land Act.
proving the reclassification of alienable and disposable land.[35] Instead, the Republic
posits that Animas affirms its right to institute reversion proceedings in instances where While the Animas ruling upholds the State's right to seek reversion with respect to
portions of forest land are erroneously included within the scope of land patents.[36] fraudulently or erroneously registered lands, it does not, in any manner, lay down the facts
Moreover, the Republic argues that in reversion proceedings, the State should not be that must be established for an action for reversion to prosper. Undoubtedly, the RTC and
made to bear the burden of proving that the land in question constitutes public domain (i.e., CA's reliance on the Animas ruling is misplaced.
forest land).[37] In any case, the Republic posits that the documentary and testimonial
evidence it had presented sufficiently proved such fact.[38] Nevertheless, such erroneous reliance on Animas, as will be discussed below, does not
advance the Republic's cause, since the principle which serves as basis for the decisions
The Petition should be denied for lack of merit. The CA did not err when it affirmed the of the RTC and CA remains correct, albeit attributed to the wrong case.
RTC Decision, as the Republic failed to establish that the Roxas Properties were classified
as forest land at the time Free Patent No. 516197 was issued. The power to classify and reclassify land lies solely with the Executive Department.
The Regalian Doctrine has long been recognized as the basic foundation of the State's
The Republic's Petition and Respondents' Compliance and Comment should be admitted property regime,[40] and has been consistently adopted under the 1935, 1973, and 1987
in the interest of substantial justice. Constitutions;[41] it espouses that all lands of the public domain belong to the State, and
At the outset, the Court notes that the parties herein, albeit at different stages of the that, as a consequence thereof, any asserted right of ownership over land necessarily
proceedings, have both prayed for the relaxation of the Rules of Court (Rules). traces back to the State.[42]

For its part, the Republic filed two (2) motions which sought for an aggregate period of At present, Section 3, Article XII of the 1987 Constitution classifies lands of the public
thirty (30) days from the expiration of the initial thirty (30)-day period prescribed by the domain into five (5) categories — forest lands, agricultural lands, timber lands, mineral
Rules for the filing of a petition for review on certiorari. The Respondents, on the other lands, and national parks. The Court's ruling in Heirs of the Late Spouses Palanca v.
hand, sought the admission of their Compliance and Comment, filed more than seven (7) Republic,[43] instructs that in the absence of any prior classification by the State,
months after the filing of the Petition.[39] unclassified lands of the public domain assume the category of forest lands not open to
disposition.[44]
Considering the nature of the issues involved in the present Petition, and the lack of
evidence showing that neither the Republic's nor the Respondents' requests for In turn, the classification of unclassified lands of the public domain, and the reclassification
accommodation had been impelled by any ill-motive, the Court resolves to admit in the of those previously classified under any of the categories set forth in the 1987 Constitution
interest of substantial justice the Republic's Petition and the Respondents' Comment with (such as the Roxas Properties), are governed by Commonwealth Act No. 141[45] dated
Compliance. November 7, 1936, otherwise known as the Public Land Act. Sections 6 and 7 thereof
provide:
The Court's ruling in Animas does not apply to the present case.
The Republic's Petition primarily proceeds from the supposition that in ruling in favor of SEC. 6. The President, upon the recommendation of the Secretary of Agriculture and
Respondents, the RTC and the CA erroneously relied on Animas. Commerce, shall from time to time classify the lands of the public domain into —

In Animas, the Republic filed an action for reversion against respondent therein, claiming (a) Alienable or disposable,
that the Free Patent issued in the latter's favor covered forest land. The Court of First (b) Timber, and
Instance dismissed the Republic's action on the ground that the original certificate of title (c) Mineral lands,
covering said land had become indefeasible, the same having been issued more than one
(1) year prior to the filing of the Republic's action. Hence, the issue brought before the and may at any time and in a like manner transfer such lands from one class to another,
Court in Animas was whether the lapse of said one (1)-year period had the effect of for the purposes of their administration and disposition.
precluding the State from initiating reversion proceedings to recover land which had been
unlawfully registered, either through fraud or oversight. Resolving the issue, the Court held SEC. 7. For the purposes of the administration and disposition of alienable or disposable
that public land fraudulently or erroneously included in the scope of patents or certificates public lands, the President, upon recommendation by the Secretary of Agriculture and
28 | P a g e
NAT RES & ENVI LAW CASES 2A_2023
Commerce, shall from time to time declare what lands are open to disposition or registered forms part of the public domain. This means that Espinosa, as the applicant,
concession under this Act. (Emphasis supplied) was able to prove by incontrovertible evidence that the property is alienable and
disposable property in the cadastral proceedings.
These provisions are clear and leave no room for interpretation - the classification and
reclassification of public lands into alienable or disposable, mineral or forest land is the xxxx
exclusive prerogative of the Executive Department,[46] and is exercised by the latter
through the President, or such other persons vested with authority to exercise the same In this case, the State, through the Solicitor General, alleges neither fraud nor
on his behalf.[47] misrepresentation in the cadastral proceedings and in the issuance of the title in
Espinosa's favor. The argument for the State is merely that the property was unlawfully
Since the power to classify and reclassify land are executive in nature, such acts, effected included in the certificate of title because it is of the public domain.
without executive authority, are void, and essentially ultra vires.
Since the case is one for reversion and not one for land registration, the burden is on the
In reversion proceedings, the State bears the burden of proving that the property in State to prove that the property was classified as timberland or forest land at the time it
question was inalienable at the time it was decreed or adjudicated in favor of the defendant. was decreed to Espinosa. To reiterate, there is no burden on [the present owner] to prove
A land registration proceeding is the manner through which an applicant confirms title to that the property in question is alienable and disposable land. At this stage, it is reasonable
real property. In this proceeding, the applicant bears the burden of overcoming the to presume that Espinosa, from whom [the present owner] derive[s] her title, had already
presumption of State ownership.[48] Accordingly, the applicant is bound to establish, established that the property is alienable and disposable land considering that she
through incontrovertible evidence, that the land sought to be registered had been declared succeeded in obtaining the OCT over it. In this reversion proceeding, the State must prove
alienable or disposable through a positive act of the State.[49] that there was an oversight or mistake in the inclusion of the property in Espinosa's title
because it was of public dominion. This is consistent with the rule that the burden of proof
Conversely, reversion proceeding is the manner through which the State seeks to revert rests on the party who, as determined by the pleadings or the nature of the case, asserts
land to the mass of the public domain;[50] it is proper when public land is fraudulently the affirmative of an issue.[56] (Emphasis and underscoring supplied)
awarded and disposed of in favor of private individuals or corporations,[51] or when a
person obtains a title under the Public Land Act which includes, by oversight, lands which Hence, to resolve this Petition, the Court must determine whether the documentary and
cannot be registered under the Torrens system as they form part of the public domain.[52] testimonial evidence offered by the Republic are sufficient to sustain its cause.

Owing to the nature of reversion proceedings and the outcome which a favorable decision The Complaint should be dismissed as the Republic failed to show that the Roxas
therein entails, the State bears the burden to prove that the land previously decreed or Properties (including Lot 1-A) were classified as forest land at the time Free Patent No.
adjudicated in favor of the defendant constitutes land which cannot be owned by private 516197 was issued in Meynardo's favor.
individuals. The Court's ruling in Republic v. Development Resources Corporation[53] is To recall, the Republic presented the following pieces of evidence to support its complaint
instructive: for reversion: (i) DENR Final Report; (ii) NAMRIA certifications; and (iii) LC Map 209.
However, these documents, whether taken individually or collectively, do not evince a
Since a complaint for reversion can upset the stability of registered titles through the positive act of reclassification by the Executive Department. As aptly stated by the CA:
cancellation of the original title and the others that emanate from it, the State bears a heavy
burden of proving the ground for its action. x x x[54] (Emphasis supplied) In this case, the Republic presented the [NAMRIA certifications], the [DENR Final Report]
and [LC Map 209] dated March 6, 1924, with an inscription that the [Roxas Properties]
Thus, in Republic v. Espinosa[55] (Espinosa), the Court held that the dismissal of the [were] reverted x x x to the category of forest land on November 24, 1949. However, it
Republic's action for reversion is proper since the Republic failed to establish that the land appears that the findings of the CENRO and the NAMRIA are based solely on such
subject thereof was classified as forest land at the time the cadastral decree in favor of the mapping [LC Map 209] where eighteen (18) hectares, including the location therein of the
defendant was issued: [Roxas Properties], [were] reclassified as forest land. Engineer [Mariano] Mendez[57]
testified that:
[I]t is undisputed that Espinosa was granted a cadastral decree and was subsequently
issued OCT No. 191-N x x x. Having been granted a decree in a cadastral proceeding, xxxx
Espinosa can be presumed to have overcome the presumption that the land sought to be
29 | P a g e
NAT RES & ENVI LAW CASES 2A_2023

Q: Q:
So you don't have the law or the order reverting that portion of land to forest land on So, you know that before a certain parcel of land would be reverted from alienable and
November 24, 1949? disposable to forest zone, there should be a basis for the same, like proclamation or law.
A: From your experience, presidential decrees?
Except only that it is a swamp land. And it is shown here in our map, sir. A:
Yes, sir. These are proclamation decrees regarding the reversion of certain land use. But
PROS. MARCO: in this particular area, the land is swamp land.

x x x [W]hat is the basis, if any, of you (sic) in declaring that this portion of land was reverted Q:
back from timber land to forest land on November 24, 1949? But in this particular case, did you encounter or did you see any law, executive order,
A: presidential proclamation declaring this parcel of land from alienable and disposable to
Our files and records. forest zone?
A:
I have not encountered any decree or presidential proclamation or order reverting this land
Q: to forest zone. x x x
What are these files and records? Even Engineer Mendez of the NAMRIA agreed that a law or proclamation is required
A: before a certain parcel of land is reclassified from alienable and disposable to forest land.
As indicated in [LC Map 209]. His insistence that because the land was (originally) swamp land that reclassification was
Engineer Mendez admitted that there was no presidential order or act reverting the made (sic), is not supported by any presidential or legal pronouncement or by practice and
classification of the subject property from alienable and disposable to forest land, thus: tradition x x x Unfortunately, the Republic failed to present any law, presidential
proclamation, order or act to prove that the subject property was indeed within the area
Q: which is reclassified as forest land. Even an administrative order from the Bureau of
Did you prepare the basis of the reversion of the land from disposable to forest land on Forestry was not presented to show that the subject property had been reclassified as
November 24, 1949? forest land.[58] (Additional emphasis and underscoring supplied)
A:
Yes, sir. The foregoing testimony, culled from the Assailed Decision, confirms that the alleged
reclassification of the Roxas Properties is bereft of basis, as it was done by Engineer
Q: Mendez on his sole account, without any prior directive from the President, or a duly
What were the basis? authorized officer from the Executive Department. In fact, the annotation appearing on LC
A: Map 209 upon which the Republic relies does not even state upon whose authority the
Yes, because when I studied that, I found out that the area was a swamp land? alleged reclassification had been made,[59] placing the annotation's validity, veracity and
worth in serious doubt.
Q:
Aside from that, that the area was a swamp land, what are your other basis? Ultimately, the Republic failed to prove that the Roxas Properties (including Lot 1-A) were
A: classified as forest land when they were decreed in Meynardo's favor in 1971. Thus, in
Nothing more, sir. As per records, that is the only basis. accordance with the Court's ruling in Development Resources Corporation and Espinosa,
the present Petition must be, as it is hereby, denied.
Q:
Did you not research any law, decree, presidential order or act as the basis of reverting WHEREFORE, premises considered, the Petition for Review on Certiorari is DENIED. The
this parcel of land to forest zone on November 24, 1949? Assailed Decision of the Court of Appeals dated July 18, 2014 and Resolution dated May
A: 20, 2015 in CA-G.R. CV No. 98120 are hereby AFFIRMED.
I have even decrees or law reverting certain area to forest land but not in this particular
area. SO ORDERED.
30 | P a g e
NAT RES & ENVI LAW CASES 2A_2023
6. vs.
SOUTHEAST MINADANAO GOLD MINING CORPORATION, Respondent.
G.R. No. 152613 & No. 152628 June 23, 2006
DECISION
APEX MINING CO., INC., petitioner,
vs. CHICO-NAZARIO, J.:
SOUTHEAST MINDANAO GOLD MINING CORP., the mines adjudication board,
provincial mining regulatory board (PMRB-DAVAO), MONKAYO INTEGRATED On 27 February 1931, Governor General Dwight F. Davis issued Proclamation No. 369,
SMALL SCALE MINERS ASSOCIATION, INC., ROSENDO VILLAFLOR, BALITE establishing the Agusan-Davao-Surigao Forest Reserve consisting of approximately
COMMUNAL PORTAL MINING COOPERATIVE, DAVAO UNITED MINERS 1,927,400 hectares.1
COOPERATIVE, ANTONIO DACUDAO, PUTING-BATO GOLD MINERS
COOPERATIVE, ROMEO ALTAMERA, THELMA CATAPANG, LUIS GALANG, The disputed area, a rich tract of mineral land, is inside the forest reserve located at
RENATO BASMILLO, FRANCISCO YOBIDO, EDUARDO GLORIA, EDWIN ASION, Monkayo, Davao del Norte, and Cateel, Davao Oriental, consisting of 4,941.6759
MACARIO HERNANDEZ, REYNALDO CARUBIO, ROBERTO BUNIALES, RUDY hectares.2 This mineral land is encompassed by Mt. Diwata, which is situated in the
ESPORTONO, ROMEO CASTILLO, JOSE REA, GIL GANADO, PRIMITIVA LICAYAN, municipalities of Monkayo and Cateel. It later became known as the "Diwalwal Gold Rush
LETICIA ALQUEZA and joel brillantes management mining Area." It has since the early 1980’s been stormed by conflicts brought about by the
corporation, Respondents. numerous mining claimants scrambling for gold that lies beneath its bosom.

x--------------------------------------x On 21 November 1983, Camilo Banad and his group, who claimed to have first discovered
traces of gold in Mount Diwata, filed a Declaration of Location (DOL) for six mining claims
G.R. No. 152619-20 June 23, 2006 in the area.

BALITE COMMUNAL PORTAL MINING COOPERATIVE, petitioner, Camilo Banad and some other natives pooled their skills and resources and organized the
vs. Balite Communal Portal Mining Cooperative (Balite).3
SOUTHEAST MINDANAO GOLD MINING CORPORATION, APEX MINING CO., INC.,
the mines adjudication board, provincial mining regulatory board (PMRB-DAVAO), On 12 December 1983, Apex Mining Corporation (Apex) entered into operating
MONKAYO INTEGRATED SMALL SCALE MINERS ASSOCIATION, INC., ROSENDO agreements with Banad and his group.
VILLAFLOR, DAVAO UNITED MINERS COOPERATIVE, ANTONIO DACUDAO,
PUTING-BATO GOLD MINERS COOPERATIVE, ROMEO ALTAMERA, THELMA
From November 1983 to February 1984, several individual applications for mining
CATAPANG, LUIS GALANG, RENATO BASMILLO, FRANCISCO YOBIDO,
locations over mineral land covering certain parts of the Diwalwal gold rush area were filed
EDUARDO GLORIA, EDWIN ASION, MACARIO HERNANDEZ, REYNALDO
with the Bureau of Mines and Geo-Sciences (BMG).
CARUBIO, ROBERTO BUNIALES, RUDY ESPORTONO, ROMEO CASTILLO, JOSE
REA, GIL GANADO, PRIMITIVA LICAYAN, LETICIA ALQUEZA and joel brillantes
management mining corporation, Respondents. On 2 February 1984, Marcopper Mining Corporation (MMC) filed 16 DOLs or mining claims
for areas adjacent to the area covered by the DOL of Banad and his group. After realizing
that the area encompassed by its mining claims is a forest reserve within the coverage of
x--------------------------------------x
Proclamation No. 369 issued by Governor General Davis, MMC abandoned the same and
instead applied for a prospecting permit with the Bureau of Forest Development (BFD).
G.R. No. 152870-71 June 23, 2006
On 1 July 1985, BFD issued a Prospecting Permit to MMC covering an area of 4,941.6759
THE MINES ADJUDICATION BOARD AND ITS MEMBERS, THE HON. VICTOR O. hectares traversing the municipalities of Monkayo and Cateel, an area within the forest
RAMOS (Chairman), UNDERSECRETARY VIRGILIO MARCELO (Member) and reserve under Proclamation No. 369. The permit embraced the areas claimed by Apex
DIRECTOR HORACIO RAMOS (Member), petitioners, and the other individual mining claimants.
31 | P a g e
NAT RES & ENVI LAW CASES 2A_2023
On 11 November 1985, MMC filed Exploration Permit Application No. 84-40 with the BMG. the Agusan-Davao-Surigao Forest Reserve as non-forest lands and open to small-scale
On 10 March 1986, the BMG issued to MCC Exploration Permit No. 133 (EP 133). mining purposes.

Discovering the existence of several mining claims and the proliferation of small-scale As DAO No. 66 declared a portion of the contested area open to small scale miners,
miners in the area covered by EP 133, MMC thus filed on 11 April 1986 before the BMG a several mining entities filed applications for Mineral Production Sharing Agreement
Petition for the Cancellation of the Mining Claims of Apex and Small Scale Mining Permit (MPSA).
Nos. (x-1)-04 and (x-1)-05 which was docketed as MAC No. 1061. MMC alleged that the
areas covered by its EP 133 and the mining claims of Apex were within an established and On 25 August 1993, Monkayo Integrated Small Scale Miners Association (MISSMA) filed
existing forest reservation (Agusan-Davao-Surigao Forest Reserve) under Proclamation an MPSA application which was denied by the BMG on the grounds that the area applied
No. 369 and that pursuant to Presidential Decree No. 463,4 acquisition of mining rights for is within the area covered by MMC EP 133 and that the MISSMA was not qualified to
within a forest reserve is through the application for a permit to prospect with the BFD and apply for an MPSA under DAO No. 82,7 Series of 1990.
not through registration of a DOL with the BMG.
On 5 January 1994, Rosendo Villaflor and his group filed before the BMG a Petition for
On 23 September 1986, Apex filed a motion to dismiss MMC’s petition alleging that its Cancellation of EP 133 and for the admission of their MPSA Application. The Petition was
mining claims are not within any established or proclaimed forest reserve, and as such, docketed as RED Mines Case No. 8-8-94. Davao United Miners Cooperative (DUMC) and
the acquisition of mining rights thereto must be undertaken via registration of DOL with the Balite intervened and likewise sought the cancellation of EP 133.
BMG and not through the filing of application for permit to prospect with the BFD.
On 16 February 1994, MMC assigned EP 133 to Southeast Mindanao Gold Mining
On 9 December 1986, BMG dismissed MMC’s petition on the ground that the area covered Corporation (SEM), a domestic corporation which is alleged to be a 100% -owned
by the Apex mining claims and MMC’s permit to explore was not a forest reservation. It subsidiary of MMC.
further declared null and void MMC’s EP 133 and sustained the validity of Apex mining
claims over the disputed area. On 14 June 1994, Balite filed with the BMG an MPSA application within the contested area
that was later on rejected.
MMC appealed the adverse order of BMG to the Department of Environment and Natural
Resources (DENR). On 23 June 1994, SEM filed an MPSA application for the entire 4,941.6759 hectares under
EP 133, which was also denied by reason of the pendency of RED Mines Case No. 8-8-
On 15 April 1987, after due hearing, the DENR reversed the 9 December 1996 order of 94. On 1 September 1995, SEM filed another MPSA application.
BMG and declared MMC’s EP 133 valid and subsisting.
On 20 October 1995, BMG accepted and registered SEM’s MPSA application and the
Apex filed a Motion for Reconsideration with the DENR which was subsequently denied. Deed of Assignment over EP 133 executed in its favor by MMC. SEM’s application was
Apex then filed an appeal before the Office of the President. On 27 July 1989, the Office designated MPSA Application No. 128 (MPSAA 128). After publication of SEM’s
of the President, through Assistant Executive Secretary for Legal Affairs, Cancio C. application, the following filed before the BMG their adverse claims or oppositions:
Garcia,5 dismissed Apex’s appeal and affirmed the DENR ruling.
a) MAC Case No. 004 (XI) – JB Management Mining Corporation;
Apex filed a Petition for Certiorari before this Court. The Petition was docketed as G.R. No.
92605 entitled, "Apex Mining Co., Inc. v. Garcia."6 On 16 July 1991, this Court rendered a b) MAC Case No. 005(XI) – Davao United Miners Cooperative;
Decision against Apex holding that the disputed area is a forest reserve; hence, the proper
procedure in acquiring mining rights therein is by initially applying for a permit to prospect
c) MAC Case No. 006(XI) – Balite Integrated Small Scale Miner’s Cooperative;
with the BFD and not through a registration of DOL with the BMG.
d) MAC Case No. 007(XI) – Monkayo Integrated Small Scale Miner’s Association,
On 27 December 1991, then DENR Secretary Fulgencio Factoran, Jr. issued Department
Inc. (MISSMA);
Administrative Order No. 66 (DAO No. 66) declaring 729 hectares of the areas covered by

32 | P a g e
NAT RES & ENVI LAW CASES 2A_2023
e) MAC Case No. 008(XI) – Paper Industries Corporation of the Philippines; extraction or removal of minerals from the ground, and that they were large-scale miners.
The decretal portion of the PA resolution pronounces:
f) MAC Case No. 009(XI) – Rosendo Villafor, et al.;
VIEWED IN THE LIGHT OF THE FOREGOING, the validity of Expoloration Permit No.
g) MAC Case No. 010(XI) – Antonio Dacudao; 133 is hereby reiterated and all the adverse claims against MPSAA No. 128 are
DISMISSED.13
h) MAC Case No. 011(XI) – Atty. Jose T. Amacio;
Undaunted by the PA ruling, the adverse claimants appealed to the Mines Adjudication
i) MAC Case No. 012(XI) – Puting-Bato Gold Miners Cooperative; Board (MAB). In a Decision dated 6 January 1998, the MAB considered erroneous the
dismissal by the PA of the adverse claims filed against MMC and SEM over a mere
technicality of failure to submit a sketch plan. It argued that the rules of procedure are not
j) MAC Case No. 016(XI) – Balite Communal Portal Mining Cooperative;
meant to defeat substantial justice as the former are merely secondary in importance to
the latter. Dealing with the question on EP 133’s validity, the MAB opined that said issue
k) MAC Case No. 97-01(XI) – Romeo Altamera, et al.8 was not crucial and was irrelevant in adjudicating the appealed case because EP 133 has
long expired due to its non-renewal and that the holder of the same, MMC, was no longer
To address the matter, the DENR constituted a Panel of Arbitrators (PA) to resolve the a claimant of the Agusan-Davao-Surigao Forest Reserve having relinquished its right to
following: SEM. After it brushed aside the issue of the validity of EP 133 for being irrelevant, the MAB
proceeded to treat SEM’s MPSA application over the disputed area as an entirely new and
(a) The adverse claims on MPSAA No. 128; and distinct application. It approved the MPSA application, excluding the area segregated by
DAO No. 66, which declared 729 hectares within the Diwalwal area as non-forest lands
(b) The Petition to Cancel EP 133 filed by Rosendo Villaflor docketed as RED Case open for small-scale mining. The MAB resolved:
No. 8-8-94.9
WHEREFORE, PREMISES CONSIDERED, the decision of the Panel of Arbitrators dated
On 13 June 1997, the PA rendered a resolution in RED Mines Case No. 8-8-94. As to the 13 June 1997 is hereby VACATED and a new one entered in the records of the case as
Petition for Cancellation of EP 133 issued to MMC, the PA relied on the ruling in Apex follows:
Mining Co., Inc. v. Garcia,10 and opined that EP 133 was valid and subsisting. It also
declared that the BMG Director, under Section 99 of the Consolidated Mines 1. SEM’s MPSA application is hereby given due course subject to the full and strict
Administrative Order implementing Presidential Decree No. 463, was authorized to issue compliance of the provisions of the Mining Act and its Implementing Rules and
exploration permits and to renew the same without limit. Regulations;

With respect to the adverse claims on SEM’s MPSAA No. 128, the PA ruled that adverse 2. The area covered by DAO 66, series of 1991, actually occupied and actively
claimants’ petitions were not filed in accordance with the existing rules and regulations mined by the small-scale miners on or before August 1, 1987 as determined by the
governing adverse claims because the adverse claimants failed to submit the sketch plan Provincial Mining Regulatory Board (PMRB), is hereby excluded from the area
containing the technical description of their respective claims, which was a mandatory applied for by SEM;
requirement for an adverse claim that would allow the PA to determine if indeed there is
an overlapping of the area occupied by them and the area applied for by SEM. It added 3. A moratorium on all mining and mining-related activities, is hereby imposed until
that the adverse claimants were not claim owners but mere occupants conducting illegal such time that all necessary procedures, licenses, permits, and other requisites as
mining activities at the contested area since only MMC or its assignee SEM had valid provided for by RA 7076, the Mining Act and its Implementing Rules and
mining claims over the area as enunciated in Apex Mining Co., Inc. v. Garcia.11 Also, it Regulations and all other pertinent laws, rules and regulations are complied with,
maintained that the adverse claimants were not qualified as small-scale miners under and the appropriate environmental protection measures and safeguards have
DENR Department Administrative Order No. 34 (DAO No. 34),12 or the Implementing been effectively put in place;
Rules and Regulation of Republic Act No. 7076 (otherwise known as the "People’s Small-
Scale Mining Act of 1991"), as they were not duly licensed by the DENR to engage in the
33 | P a g e
NAT RES & ENVI LAW CASES 2A_2023
4. Consistent with the spirit of RA 7076, the Board encourages SEM and all small- The Court of Appeals also faulted the DENR Secretary in implementing DAO No. 66 when
scale miners to continue to negotiate in good faith and arrive at an agreement he awarded the 729 hectares segregated from the coverage area of EP 133 to other
beneficial to all. In the event of SEM’s strict and full compliance with all the corporations who were not qualified as small-scale miners under Republic Act No. 7076.
requirements of the Mining Act and its Implementing Rules and Regulations, and
the concurrence of the small-scale miners actually occupying and actively mining As to the petitions of Villaflor and company, the Court of Appeals argued that their failure
the area, SEM may apply for the inclusion of portions of the areas segregated to submit the sketch plan to the PA, which is a jurisdictional requirement, was fatal to their
under paragraph 2 hereof, to its MPSA application. In this light, subject to the appeal. It likewise stated the Villaflor and company’s mining claims, which were based on
preceding paragraph, the contract between JB [JB Management Mining their alleged rights under DAO No. 66, cannot stand as DAO No. 66 was null and void.
Corporation] and SEM is hereby recognized.14 The dispositive portion of the Decision decreed:

Dissatisfied, the Villaflor group and Balite appealed the decision to this Court. SEM, WHEREFORE, premises considered, the Petition of Southeast Mindanao Gold Mining
aggrieved by the exclusion of 729 hectares from its MPSA application, likewise appealed. Corporation is GRANTED while the Petition of Rosendo Villaflor, et al., is DENIED for lack
Apex filed a Motion for Leave to Admit Petition for Intervention predicated on its right to of merit. The Decision of the Panel of Arbitrators dated 13 June 1997 is AFFIRMED in toto
stake its claim over the Diwalwal gold rush which was granted by the Court. These cases, and the assailed MAB Decision is hereby SET ASIDE and declared as NULL and VOID.16
however, were remanded to the Court of Appeals for proper disposition pursuant to Rule
43 of the 1997 Rules of Civil Procedure. The Court of Appeals consolidated the remanded Hence, the instant Petitions for Review on Certiorari under Rule 45 of the Rules of Court
cases as CA-G.R. SP No. 61215 and No. 61216. filed by Apex, Balite and MAB.

In the assailed Decision15 dated 13 March 2002, the Court of Appeals affirmed in toto the During the pendency of these Petitions, President Gloria Macapagal-Arroyo issued
decision of the PA and declared null and void the MAB decision. Proclamation No. 297 dated 25 November 2002. This proclamation excluded an area of
8,100 hectares located in Monkayo, Compostela Valley, and proclaimed the same as
The Court of Appeals, banking on the premise that the SEM is the agent of MMC by virtue mineral reservation and as environmentally critical area. Subsequently, DENR
of its assignment of EP 133 in favor of SEM and the purported fact that SEM is a 100% Administrative Order No. 2002-18 was issued declaring an emergency situation in the
subsidiary of MMC, ruled that the transfer of EP 133 was valid. It argued that since SEM Diwalwal gold rush area and ordering the stoppage of all mining operations therein.
is an agent of MMC, the assignment of EP 133 did not violate the condition therein Thereafter, Executive Order No. 217 dated 17 June 2003 was issued by the President
prohibiting its transfer except to MMC’s duly designated agent. Thus, despite the non- creating the National Task Force Diwalwal which is tasked to address the situation in the
renewal of EP 133 on 6 July 1994, the Court of Appeals deemed it relevant to declare EP Diwalwal Gold Rush Area.
133 as valid since MMC’s mining rights were validly transferred to SEM prior to its
expiration. In G.R. No. 152613 and No. 152628, Apex raises the following issues:

The Court of Appeals also ruled that MMC’s right to explore under EP 133 is a property I
right which the 1987 Constitution protects and which cannot be divested without the
holder’s consent. It stressed that MMC’s failure to proceed with the extraction and
WHETHER OR NOT SOUTHEAST MINDANAO GOLD MINING’S [SEM] E.P. 133 IS
utilization of minerals did not diminish its vested right to explore because its failure was
NULL AND VOID DUE TO THE FAILURE OF MARCOPPER TO COMPLY WITH THE
not attributable to it.
TERMS AND CONDITIONS PRESCRIBED IN EP 133.
Reading Proclamation No. 369, Section 11 of Commonwealth Act 137, and Sections 6, 7,
II
and 8 of Presidential Decree No. 463, the Court of Appeals concluded that the issuance
of DAO No. 66 was done by the DENR Secretary beyond his power for it is the President
who has the sole power to withdraw from the forest reserve established under WHETHER OR NOT APEX HAS A SUPERIOR AND PREFERENTIAL RIGHT TO STAKE
Proclamation No. 369 as non-forest land for mining purposes. Accordingly, the segregation IT’S CLAIM OVER THE ENTIRE 4,941 HECTARES AGAINST SEM AND THE OTHER
of 729 hectares of mining areas from the coverage of EP 133 by the MAB was unfounded. CLAIMANTS PURSUANT TO THE TIME-HONORED PRINCIPLE IN MINING LAW THAT
"PRIORITY IN TIME IS PRIORITY IN RIGHT."17

34 | P a g e
NAT RES & ENVI LAW CASES 2A_2023
In G.R. No. 152619-20, Balite anchors its petition on the following grounds: II. Whether or not the Court of Appeals erred in declaring that the DENR Secretary
has no authority to issue DAO No. 66; and
I
III. Whether or not the subsequent acts of the executive department such as the
WHETHER OR NOT THE MPSA OF SEM WHICH WAS FILED NINE (9) DAYS LATE issuance of Proclamation No. 297, and DAO No. 2002-18 can outweigh Apex and
(JUNE 23, 1994) FROM THE FILING OF THE MPSA OF BALITE WHICH WAS FILED ON Balite’s claims over the Diwalwal Gold Rush Area.
JUNE 14, 1994 HAS A PREFERENTIAL RIGHT OVER THAT OF BALITE.
On the first issue, Apex takes exception to the Court of Appeals’ ruling upholding the
II validity of MMC’s EP 133 and its subsequent transfer to SEM asserting that MMC failed to
comply with the terms and conditions in its exploration permit, thus, MMC and its
WHETHER OR NOT THE DISMISSAL BY THE PANEL OF ARBITRATORS OF THE successor-in-interest SEM lost their rights in the Diwalwal Gold Rush Area. Apex pointed
ADVERSE CLAIM OF BALITE ON THE GROUND THAT BALITE FAILED TO SUBMIT out that MMC violated four conditions in its permit. First, MMC failed to comply with the
THE REQUIRED SKETCH PLAN DESPITE THE FACT THAT BALITE, HAD IN FACT mandatory work program, to complete exploration work, and to declare a mining feasibility.
SUBMITTED ON TIME WAS A VALID DISMISSAL OF BALITE’S ADVERSE CLAIM. Second, it reneged on its duty to submit an Environmental Compliance Certificate. Third,
it failed to comply with the reportorial requirements. Fourth, it violated the terms of EP 133
when it assigned said permit to SEM despite the explicit proscription against its transfer.
III
Apex likewise emphasizes that MMC failed to file its MPSA application required under
WHETHER OR NOT THE ACTUAL OCCUPATION AND SMALL-MINING OPERATIONS
DAO No. 8220 which caused its exploration permit to lapse because DAO No. 82 mandates
OF BALITE PURSUANT TO DAO 66 IN THE 729 HECTARES WHICH WAS PART OF
holders of exploration permits to file a Letter of Intent and a MPSA application not later
THE 4,941.6759 HECTARES COVERED BY ITS MPSA WHICH WAS REJECTED BY
than 17 July 1991. It said that because EP 133 expired prior to its assignment to SEM,
THE BUREAU OF MINES AND GEOSCIENCES WAS ILLEGAL.18
SEM’s MPSA application should have been evaluated on its own merit.
In G.R. No. 152870-71, the MAB submits two issues, to wit:
As regards the Court of Appeals recognition of SEM’s vested right over the disputed area,
Apex bewails the same to be lacking in statutory bases. According to Apex, Presidential
I Decree No. 463 and Republic Act No. 7942 impose upon the claimant the obligation of
actually undertaking exploration work within the reserved lands in order to acquire priority
WHETHER OR NOT EP NO. 133 IS STILL VALID AND SUBSISTING. right over the area. MMC, Apex claims, failed to conduct the necessary exploration work,
thus, MMC and its successor-in-interest SEM lost any right over the area.
II
In its Memorandum, Balite maintains that EP 133 of MMC, predecessor-in-interest of SEM,
WHETHER OR NOT THE SUBSEQUENT ACTS OF THE GOVERNMENT SUCH AS THE is an expired and void permit which cannot be made the basis of SEM’s MPSA application.
ISSUANCE OF DAO NO. 66, PROCLAMATION NO. 297, AND EXECUTIVE ORDER 217
CAN OUTWEIGH EP NO. 133 AS WELL AS OTHER ADVERSE CLAIMS OVER THE Similarly, the MAB underscores that SEM did not acquire any right from MMC by virtue of
DIWALWAL GOLD RUSH AREA.19 the transfer of EP 133 because the transfer directly violates the express condition of the
exploration permit stating that "it shall be for the exclusive use and benefit of the permittee
The common issues raised by petitioners may be summarized as follows: or his duly authorized agents." It added that while MMC is the permittee, SEM cannot be
considered as MMC’s duly designated agent as there is no proof on record authorizing
I. Whether or not the Court of Appeals erred in upholding the validity and SEM to represent MMC in its business dealings or undertakings, and neither did SEM
continuous existence of EP 133 as well as its transfer to SEM; pursue its interest in the permit as an agent of MMC. According to the MAB, the assignment
by MMC of EP 133 in favor of SEM did not make the latter the duly authorized agent of
MMC since the concept of an agent under EP 133 is not equivalent to the concept of
assignee. It finds fault in the assignment of EP 133 which lacked the approval of the DENR
35 | P a g e
NAT RES & ENVI LAW CASES 2A_2023
Secretary in contravention of Section 25 of Republic Act No. 794221 requiring his approval 1. That the permittee shall abide by the work program submitted with the
for a valid assignment or transfer of exploration permit to be valid. application or statements made later in support thereof, and which shall be
considered as conditions and essential parts of this permit;
SEM, on the other hand, counters that the errors raised by petitioners Apex, Balite and the
MAB relate to factual and evidentiary matters which this Court cannot inquire into in an 2. That permittee shall maintain a complete record of all activities and accounting
appeal by certiorari. of all expenditures incurred therein subject to periodic inspection and verification
at reasonable intervals by the Bureau of Mines at the expense of the applicant;
The established rule is that in the exercise of the Supreme Court’s power of review, the
Court not being a trier of facts, does not normally embark on a re-examination of the 3. That the permittee shall submit to the Director of Mines within 15 days after the
evidence presented by the contending parties during the trial of the case considering that end of each calendar quarter a report under oath of a full and complete statement
the findings of facts of the Court of Appeals are conclusive and binding on the Court.22 This of the work done in the area covered by the permit;
rule, however, admits of exceptions as recognized by jurisprudence, to wit:
4. That the term of this permit shall be for two (2) years to be effective from this
(1) [w]hen the findings are grounded entirely on speculation, surmises or conjectures; (2) date, renewable for the same period at the discretion of the Director of Mines and
when the inference made is manifestly mistaken, absurd or impossible; (3) when there is upon request of the applicant;
grave abuse of discretion; (4) when the judgment is based on misapprehension of facts;
(5) when the findings of facts are conflicting; (6) when in making its findings the Court of 5. That the Director of Mines may at any time cancel this permit for violation of its
Appeals went beyond the issues of the case, or its findings are contrary to the admissions provision or in case of trouble or breach of peace arising in the area subject hereof
of both the appellant and the appellee; (7) when the findings are contrary to the trial court; by reason of conflicting interests without any responsibility on the part of the
(8) when the findings are conclusions without citation of specific evidence on which they government as to expenditures for exploration that might have been incurred, or
are based; (9) when the facts set forth in the petition as well as in the petitioner’s main and as to other damages that might have been suffered by the permittee; and
reply briefs are not disputed by the respondent; (10) when the findings of fact are premised
on the supposed absence of evidence and contradicted by the evidence on record; and 6. That this permit shall be for the exclusive use and benefit of the permittee or his
(11) when the Court of Appeals manifestly overlooked certain relevant facts not disputed duly authorized agents and shall be used for mineral exploration purposes only
by the parties, which, if properly considered, would justify a different conclusion.23 and for no other purpose.

Also, in the case of Manila Electric Company v. Benamira,24 the Court in a Petition for Under Section 9027 of Presidential Decree No. 463, the applicable statute during the
Review on Certiorari, deemed it proper to look deeper into the factual circumstances of issuance of EP 133, the DENR Secretary, through Director of BMG, is charged with
the case since the Court of Appeal’s findings are at odds to those of the National Labor carrying out the said law. Also, under Commonwealth Act No. 136, also known as "An Act
Relations Commission (NLRC). Just like in the foregoing case, it is this Court’s considered Creating The Bureau of Mines," which was approved on 7 November 1936, the Director of
view that a re-evaluation of the attendant facts surrounding the present case is appropriate Mines has the direct charge of the administration of the mineral lands and minerals, and
considering that the findings of the MAB are in conflict with that of the Court of Appeals. of the survey, classification, lease or any other form of concession or disposition thereof
under the Mining Act.28 This power of administration includes the power to prescribe terms
I and conditions in granting exploration permits to qualified entities. Thus, in the grant of EP
133 in favor of the MMC, the Director of the BMG acted within his power in laying down
At the threshold, it is an undisputed fact that MMC assigned to SEM all its rights under EP the terms and conditions attendant thereto.
133 pursuant to a Deed of Assignment dated 16 February 1994. 25
Condition number 6 categorically states that the permit shall be for the exclusive use and
EP 133 is subject to the following terms and conditions26 : benefit of MMC or its duly authorized agents. While it may be true that SEM, the assignee
of EP 133, is a 100% subsidiary corporation of MMC, records are bereft of any evidence
showing that the former is the duly authorized agent of the latter. For a contract of agency
to exist, it is essential that the principal consents that the other party, the agent, shall act
on its behalf, and the agent consents so as to act.29 In the case of Yu Eng Cho v. Pan
36 | P a g e
NAT RES & ENVI LAW CASES 2A_2023
American World Airways, Inc.,30 this Court had the occasion to set forth the elements of rights and obligations MMC have under the permit in favor of SEM, thus, making SEM the
agency, viz: permittee. It is not a mere grant of authority to SEM, as an agent of MMC, to use the permit.
It is a total abdication of MMC’s rights over the permit. Hence, the assignment in question
(1) consent, express or implied, of the parties to establish the relationship; did not make SEM the authorized agent of MMC to make use and benefit from EP 133.

(2) the object is the execution of a juridical act in relation to a third person; The condition stipulating that the permit is for the exclusive use of the permittee or its duly
authorized agent is not without any reason. Exploration permits are strictly granted to
(3) the agent acts as a representative and not for himself; entities or individuals possessing the resources and capability to undertake mining
operations. Without such a condition, non-qualified entities or individuals could circumvent
the strict requirements under the law by the simple expediency acquiring the permit from
(4) the agent acts within the scope of his authority.
the original permittee.
The existence of the elements of agency is a factual matter that needs to be established
We cannot lend recognition to the Court of Appeals’ theory that SEM, being a 100%
or proven by evidence. The burden of proving that agency is extant in a certain case rests
subsidiary of MMC, is automatically an agent of MMC.
in the party who sets forth such allegation. This is based on the principle that he who
alleges a fact has the burden of proving it.31 It must likewise be emphasized that the
evidence to prove this fact must be clear, positive and convincing.32 A corporation is an artificial being created by operation of law, having the right of
succession and the powers, attributes, and properties expressly authorized by law or
incident to its existence.36 It is an artificial being invested by law with a personality separate
In the instant Petitions, it is incumbent upon either MMC or SEM to prove that a contract
and distinct from those of the persons composing it as well as from that of any other legal
of agency actually exists between them so as to allow SEM to use and benefit from EP
entity to which it may be related.37 Resultantly, absent any clear proof to the contrary, SEM
133 as the agent of MMC. SEM did not claim nor submit proof that it is the designated
is a separate and distinct entity from MMC.
agent of MMC to represent the latter in its business dealings or undertakings. SEM cannot,
therefore, be considered as an agent of MMC which can use EP 133 and benefit from it.
Since SEM is not an authorized agent of MMC, it goes without saying that the assignment The Court of Appeals pathetically invokes the doctrine of piercing the corporate veil to
or transfer of the permit in favor of SEM is null and void as it directly contravenes the terms legitimize the prohibited transfer or assignment of EP 133. It stresses that SEM is just a
and conditions of the grant of EP 133. business conduit of MMC, hence, the distinct legal personalities of the two entities should
not be recognized. True, the corporate mask may be removed when the corporation is just
an alter ego or a mere conduit of a person or of another corporation. 38 For reasons of
Furthermore, the concept of agency is distinct from assignment. In agency, the agent acts
public policy and in the interest of justice, the corporate veil will justifiably be impaled only
not on his own behalf but on behalf of his principal.33 While in assignment, there is total
when it becomes a shield for fraud, illegality or inequity committed against a third
transfer or relinquishment of right by the assignor to the assignee.34 The assignee takes
person.39 However, this Court has made a caveat in the application of the doctrine of
the place of the assignor and is no longer bound to the latter. The deed of assignment
piercing the corporate veil. Courts should be mindful of the milieu where it is to be applied.
clearly stipulates:
Only in cases where the corporate fiction was misused to such an extent that injustice,
fraud or crime was committed against another, in disregard of its rights may the veil be
1. That for ONE PESO (P1.00) and other valuable consideration received by the pierced and removed. Thus, a subsidiary corporation may be made to answer for the
ASSIGNOR from the ASSIGNEE, the ASSIGNOR hereby ASSIGNS, TRANSFERS and liabilities and/or illegalities done by the parent corporation if the former was organized for
CONVEYS unto the ASSIGNEE whatever rights or interest the ASSIGNOR may have in the purpose of evading obligations that the latter may have entered into. In other words,
the area situated in Monkayo, Davao del Norte and Cateel, Davao Oriental, identified as this doctrine is in place in order to expose and hold liable a corporation which commits
Exploration Permit No. 133 and Application for a Permit to Prospect in Bunawan, Agusan illegal acts and use the corporate fiction to avoid liability from the said acts. The doctrine
del Sur respectively.35 of piercing the corporate veil cannot therefore be used as a vehicle to commit prohibited
acts because these acts are the ones which the doctrine seeks to prevent.
Bearing in mind the just articulated distinctions and the language of the Deed of
Assignment, it is readily obvious that the assignment by MMC of EP 133 in favor of SEM To our mind, the application of the foregoing doctrine is unwarranted. The assignment of
did not make the latter the former’s agent. Such assignment involved actual transfer of all the permit in favor of SEM is utilized to circumvent the condition of non-transferability of
37 | P a g e
NAT RES & ENVI LAW CASES 2A_2023
the exploration permit. To allow SEM to avail itself of this doctrine and to approve the extended for 12 months or until 6 July 1994.41 MMC never renewed its permit prior and
validity of the assignment is tantamount to sanctioning illegal act which is what the doctrine after its expiration. Thus, EP 133 expired by non-renewal.
precisely seeks to forestall.
With the expiration of EP 133 on 6 July 1994, MMC lost any right to the Diwalwal Gold
Quite apart from the above, a cursory consideration of the mining law pertinent to the case, Rush Area. SEM, on the other hand, has not acquired any right to the said area because
will, indeed, demonstrate the infraction committed by MMC in its assignment of EP 133 to the transfer of EP 133 in its favor is invalid. Hence, both MMC and SEM have not acquired
SEM. any vested right over the 4,941.6759 hectares which used to be covered by EP 133.

Presidential Decree No. 463, enacted on 17 May 1974, otherwise known as the Mineral II
Resources Development Decree, which governed the old system of exploration,
development, and utilization of mineral resources through "license, concession or lease" The Court of Appeals theorizes that DAO No. 66 was issued beyond the power of the
prescribed: DENR Secretary since the power to withdraw lands from forest reserves and to declare
the same as an area open for mining operation resides in the President.
SEC. 97. Assignment of Mining Rights. – A mining lease contract or any interest
therein shall not be transferred, assigned, or subleased without the prior approval of the Under Proclamation No. 369 dated 27 February 1931, the power to convert forest reserves
Secretary: Provided, That such transfer, assignment or sublease may be made only to a as non-forest reserves is vested with the DENR Secretary. Proclamation No. 369 partly
qualified person possessing the resources and capability to continue the mining operations states:
of the lessee and that the assignor has complied with all the obligations of the lease:
Provided, further, That such transfer or assignment shall be duly registered with the office From this reserve shall be considered automatically excluded all areas which had already
of the mining recorder concerned. (Emphasis supplied.) been certified and which in the future may be proclaimed as classified and certified lands
and approved by the Secretary of Agriculture and Natural Resources.42
The same provision is reflected in Republic Act No. 7942, otherwise known as the
Philippine Mining Act of 1995, which is the new law governing the exploration, However, a subsequent law, Commonwealth Act No. 137, otherwise known as "The Mining
development and utilization of the natural resources, which provides: Act" which was approved on 7 November 1936 provides:

SEC. 25. Transfer or Assignment. - An exploration permit may be transferred or assigned Sec. 14. Lands within reservations for purposes other than mining, which, after such
to a qualified person subject to the approval of the Secretary upon the recommendation of reservation is made, are found to be more valuable for their mineral contents than for the
the Director. purpose for which the reservation was made, may be withdrawn from such reservations
by the President with the concurrence of the National Assembly, and thereupon such lands
The records are bereft of any indication that the assignment bears the imprimatur of the shall revert to the public domain and be subject to disposition under the provisions of this
Secretary of the DENR. Presidential Decree No. 463, which is the governing law when the Act.
assignment was executed, explicitly requires that the transfer or assignment of mining
rights, including the right to explore a mining area, must be with the prior approval of the Unlike Proclamation No. 369, Commonwealth Act No. 137 vests solely in the President,
Secretary of DENR. Quite conspicuously, SEM did not dispute the allegation that the Deed with the concurrence of the National Assembly, the power to withdraw forest reserves
of Assignment was made without the prior approval of the Secretary of DENR. Absent the found to be more valuable for their mineral contents than for the purpose for which the
prior approval of the Secretary of DENR, the assignment of EP 133, was, therefore, without reservation was made and convert the same into non-forest reserves. A similar provision
legal effect for violating the mandatory provision of Presidential Decree No. 463. can also be found in Presidential Decree No. 463 dated 17 May 1974, with the
modifications that (1) the declaration by the President no longer requires the concurrence
An added significant omission proved fatal to MMC/SEM’s cause. While it is true that the of the National Assembly and (2) the DENR Secretary merely exercises the power to
case of Apex Mining Co., Inc. v. Garcia40 settled the issue of which between Apex and recommend to the President which forest reservations are to be withdrawn from the
MMC validly acquired mining rights over the disputed area, such rights, though, had been coverage thereof. Section 8 of Presidential Decree No. 463 reads:
extinguished by subsequent events. Records indicate that on 6 July 1993, EP 133 was

38 | P a g e
NAT RES & ENVI LAW CASES 2A_2023
SEC. 8. Exploration and Exploitation of Reserved Lands. – When lands within reservations, WHEREAS, it is in the national interest to prevent the further degradation of the
which have been established for purposes other than mining, are found to be more environment and to resolve the health and peace and order problems spawned by the
valuable for their mineral contents, they may, upon recommendation of the Secretary be unregulated mining operations in the said area;
withdrawn from such reservation by the President and established as a mineral reservation.
WHEREAS, these problems may be effectively addressed by rationalizing mining
Against the backdrop of the applicable statutes which govern the issuance of DAO No. 66, operations in the area through the establishment of a mineral reservation;
this Court is constrained to rule that said administrative order was issued not in accordance
with the laws. Inescapably, DAO No. 66, declaring 729 hectares of the areas covered by WHEREAS, after giving due notice, the Director of Mines and Geoxciences conducted
the Agusan-Davao-Surigao Forest Reserve as non-forest land open to small-scale mining public hearings on September 6, 9 and 11, 2002 to allow the concerned sectors and
operations, is null and void as, verily, the DENR Secretary has no power to convert forest communities to air their views regarding the establishment of a mineral reservation in the
reserves into non-forest reserves. place in question;

III WHEREAS, pursuant to the Philippine Mining Act of 1995 (RA 7942), the President may,
upon the recommendation of the Director of Mines and Geosciences, through the
It is the contention of Apex that its right over the Diwalwal gold rush area is superior to that Secretary of Environment and Natural Resources, and when the national interest so
of MMC or that of SEM because it was the first one to occupy and take possession of the requires, establish mineral reservations where mining operations shall be undertaken by
area and the first to record its mining claims over the area. the Department directly or thru a contractor;

For its part, Balite argues that with the issuance of DAO No. 66, its occupation in the WHEREAS, as a measure to attain and maintain a rational and orderly balance between
contested area, particularly in the 729 hectares small-scale mining area, has entitled it to socio-economic growth and environmental protection, the President may, pursuant to
file its MPSA. Balite claims that its MPSA application should have been given preference Presidential Decree No. 1586, as amended, proclaim and declare certain areas in the
over that of SEM because it was filed ahead. country as environmentally critical;

The MAB, on the other hand, insists that the issue on who has superior right over the NOW, THEREFORE, I, GLORIA MACAPAGAL-ARROYO, President of the Philippines,
disputed area has become moot and academic by the supervening events. By virtue of upon recommendation of the Secretary of the Department of Environment and Natural
Proclamation No. 297 dated 25 November 2002, the disputed area was declared a mineral Resources (DENR), and by virtue of the powers vested in me by law, do hereby exclude
reservation. certain parcel of land located in Monkayo, Compostela Valley, and proclaim the same as
mineral reservation and as environmentally critical area, with metes and bound as defined
Proclamation No. 297 excluded an area of 8,100 hectares located in Monkayo, by the following geographical coordinates;
Compostela Valley, and proclaimed the same as mineral reservation and as
environmentally critical area, viz: xxxx

WHEREAS, by virtue of Proclamation No. 369, series of 1931, certain tracts of public land with an area of Eight Thousand One Hundred (8,100) hectares, more or less. Mining
situated in the then provinces of Davao, Agusan and Surigao, with an area of operations in the area may be undertaken either by the DENR directly, subject to payment
approximately 1,927,400 hectares, were withdrawn from settlement and disposition, of just compensation that may be due to legitimate and existing claimants, or thru a
excluding, however, those portions which had been certified and/or shall be classified and qualified contractor, subject to existing rights, if any.
certified as non-forest lands;
The DENR shall formulate and issue the appropriate guidelines, including the
WHEREAS, gold deposits have been found within the area covered by Proclamation No. establishment of an environmental and social fund, to implement the intent and provisions
369, in the Municipality of Monkayo, Compostela Valley Province, and unregulated small of this Proclamation.
to medium-scale mining operations have, since 1983, been undertaken therein, causing in
the process serious environmental, health, and peace and order problems in the area;

39 | P a g e
NAT RES & ENVI LAW CASES 2A_2023
Upon the effectivity of the 1987 Constitution, the State assumed a more dynamic role in control of the State through the executive branch. Pursuant to Section 5 of Republic Act
the exploration, development and utilization of the natural resources of the country. 43 With No. 7942, the State can either directly undertake the exploration, development and
this policy, the State may pursue full control and supervision of the exploration, utilization of the area or it can enter into agreements with qualified entities, viz:
development and utilization of the country’s natural mineral resources. The options open
to the State are through direct undertaking or by entering into co-production, joint venture, SEC 5. Mineral Reservations. – When the national interest so requires, such as when there
or production-sharing agreements, or by entering into agreement with foreign-owned is a need to preserve strategic raw materials for industries critical to national development,
corporations for large-scale exploration, development and utilization.44 Thus, Article XII, or certain minerals for scientific, cultural or ecological value, the President may establish
Section 2, of the 1987 Constitution, specifically states: mineral reservations upon the recommendation of the Director through the
Secretary. Mining operations in existing mineral reservations and such other reservations
SEC. 2. All lands of the public domain, waters, minerals, coal, petroleum, and other mineral as may thereafter be established, shall be undertaken by the Department or through a
oils, all forces of potential energy, fisheries, forests or timber, wildlife, flora and fauna, and contractor x x x .
other natural resources are owned by the State. With the exception of agricultural lands,
all other natural resources shall not be alienated. The exploration, development, and It is now up to the Executive Department whether to take the first option, i.e., to undertake
utilization of natural resources shall be under the full control and supervision of the directly the mining operations of the Diwalwal Gold Rush Area. As already ruled, the State
State. The State may directly undertake such activities, or it may enter into co-production, may not be precluded from considering a direct takeover of the mines, if it is the only
joint venture, or production-sharing agreements with Filipino citizens, or corporations or plausible remedy in sight to the gnawing complexities generated by the gold rush. The
associations at least sixty per centum of whose capital is owned by such citizens. Such State need be guided only by the demands of public interest in settling on this option, as
agreements may be for a period not exceeding twenty-five years, renewable for not more well as its material and logistic feasibility.45 The State can also opt to award mining
than twenty-five years, and under such terms and conditions as may be provided by law. operations in the mineral reservation to private entities including petitioners Apex and
xxx Balite, if it wishes. The exercise of this prerogative lies with the Executive Department over
which courts will not interfere.
xxxx
WHEREFORE, premises considered, the Petitions of Apex, Balite and the MAB are
The President may enter into agreements with foreign-owned corporations involving either PARTIALLY GRANTED, thus:
technical or financial assistance for large-scale exploration, development, and utilization
of minerals, petroleum, and other mineral oils according to the general terms and 1. We hereby REVERSE and SET ASIDE the Decision of the Court of Appeals,
conditions provided by law, based on real contributions to the economic growth and dated 13 March 2002, and hereby declare that EP 133 of MMC has EXPIRED on
general welfare of the country. x x x (Underscoring supplied.) 7 July 1994 and that its subsequent transfer to SEM on 16 February 1994 is VOID.

Recognizing the importance of the country’s natural resources, not only for national 2. We AFFIRM the finding of the Court of Appeals in the same Decision declaring
economic development, but also for its security and national defense, Section 5 of DAO No. 66 illegal for having been issued in excess of the DENR Secretary’s
Republic Act No. 7942 empowers the President, when the national interest so requires, to authority.
establish mineral reservations where mining operations shall be undertaken directly by the
State or through a contractor. Consequently, the State, should it so desire, may now award mining operations in the
disputed area to any qualified entity it may determine. No costs.
To implement the intent and provisions of Proclamation No. 297, the DENR Secretary
issued DAO No. 2002-18 dated 12 August 2002 declaring an emergency situation in the SO ORDERED.
Diwalwal Gold Rush Area and ordering the stoppage of all mining operations therein.

The issue on who has priority right over the disputed area is deemed overtaken by the
above subsequent developments particularly with the issuance of Proclamation 297 and
DAO No. 2002-18, both being constitutionally-sanctioned acts of the Executive Branch.
Mining operations in the Diwalwal Mineral Reservation are now, therefore, within the full
40 | P a g e
NAT RES & ENVI LAW CASES 2A_2023
But you undoubtedly would understand their fears. It was their feeling that
7. entry of the government into their line of business would certainly spell for
them financial ruin as it would put into serious doubt the viability of the
G.R. No. 86953 November 6, 1990 entire marine radio communications industry. They say that, as it is today,
the industry is not viable enough. What more, they ask, if the government
MARINE RADIO COMMUNICATIONS ASSOCIATION OF THE PHILIPPINES, INC. steps in and eventually dips its strong fingers into the pie? 6
(MARCAPI), ROBERTO GAYA, DAVID ZAFRA and SEGUNDO P. LUSTRE,
JR., petitioners, xxx xxx xxx
vs.
HON. RAINERIO O. REYES, in his capacity as Secretary of the Department of On August 17, 1988, the Secretary forwarded a reply, denying Atty. Cabigao's request, for
Transportation and Communications (DOTC), HON. JOSE LUIS ALCUAZ, as the following reasons:
Commissioner of the National Telecommunications Commission (NTC), and HON.
ROSAURO SIBAL, as Chief of the Telecommunications Office (TELOF) of xxx xxx xxx
DOTC, respondents.
MARCAPI's main business concern is public correspondence. This means
F. Reyes Cabigao for petitioners. that MARCAPI handles only correspondence between passengers or crew
on board ship and their respective offices or residences. On the other hand,
the Maritime Coastal Communications System Project to be implemented
by 1989 will offer services in watch and distress signal, medical and
SARMIENTO, J.: meteorological services, port services, and public correspondence, in their
order of priority.
The petitioners are self-described "Filipino enterpreneurs deeply involved in the business
of marine radio communications in the country. 1 They are also operators of "shore-to-ship You will note that public correspondence is only fourth in the order of
and ship-to-shore public marine coastal radio stations, 2 and are holders of certificates of priority of services to be offered by the present maritime project. Primarily,
public convenience duly issued by the National Telecommunications Commission. Among it will offer distress and safety communications service which is obligatory
other things, they handle correspondence between vessel passengers or crew and the in the maritime mobile service. This consists of monitoring by coast
public. 3 stations of distress signal from ships in trouble and relaying the messages
to the Philippine Coast Guard which will undertake the search and rescue
Sometime in July, 1988, the Department of Transportation and Communications unveiled operations. It also includes safety communication which refers to weather
an P880-million maritime coastal communications system project, designed to "ensure broadcast and typhoon signals that will be broadcast by the coast stations
safety of lives at sea (SOLAS) through the establishment of efficient communication regularly. These services are offered to the public for free.
facilities between coast stations and ship stations and the improvement of safety in
navigational routes at sea." 4 It was set out to provide, among other things, ship-to- shore It is worth noting, as it is significant, that the confidence of the public in the
and shore-to-ship public corresponding, free of charge. 5 competence of private firms to carry out the aforecited objectives has
already been eroded. After that tragic incident of the sinking of MV Dona
On August 1, 1988, Atty. F. Reyes Cabigao, in his capacity as counsel for the petitioner, Paz, the National Telecommunications Commission and MARINA
Marine Radio Communications Association of the Philippines, Inc., addressed an appeal conducted constant monitoring by sending distress signals. Out of 1,000
to then Secretary Rainerio Reyes, in the tenor as follows: licensed private operators only one (1) responded to the signal. 7

xxx xxx xxx On February 20, 1989, the petitioners brought the instant suit, alleging, in essence, that
Secretary Rainerio Reyes had been guilty of a grave abuse of discretion.

41 | P a g e
NAT RES & ENVI LAW CASES 2A_2023
On June 7, 1990, the Court issued a Resolution, in view of the departure of Secretary The petitioners can not legitimately rely on the provisions of Section 20, of Article II, of the
Rainerio Reyes, requiring the present incumbent, Secretary Oscar Orbos, to inform the Constitution, to defeat the act complained of. The mandate "recogni[zing] the
Court whether or not the Department is adopting the action of Secretary Reyes. On August indispensable role of the private sector" is no more than an acknowledgment of the
16, 1990, Assistant Secretary Wilfredo Trinidad informed us that Secretary Orbos is importance of private initiative in building the nation. However, it is not a call for official
adopting the action complained of. abdication of duty to citizenry.

The petitioners hold that the Department can not compete in the business of public The novel provisions of the Charter prescribing private sector participation, especially in
correspondence, and rely on the provisions of Section 20, of Article II, of the Constitution, the field of economic activity, 13 come, indeed, no more as responses to State monopoly
which states: of economic forces which has unfairly kept individual initiative from the economic
processes and has held back competitiveness in the market. The Constitution does not
Sec. 20. The State recognizes the indispensable role of the private sector, bar, however, the Government from undertaking its own initiatives, especially in the
encourages private enterprise, and provides incentives to needed domain of public service, and neither does it repudiate its primacy as chief economic
investments. caretaker of the nation.

The Solicitor General, on the other hand, submits that in spite of the above provision, the The principle of laissez faire has long been denied validity in this jurisdiction. In 1969, the
Government "cannot abandon its ministerial functions of rendering public services to the Court promulgated Agricultural Credit and Cooperative Financing Administration v.
citizenry which private capital would not ordinarily undertake, or which by its very nature is Confederation of Unions in Government Corporations and offices, 14 where it was held:
better equipped to administer for the public welfare than by any private individual or entity. 8
xxx xxx xxx
There is no merit in this petition.
... The areas which used to be left to private enterprise and initiative and
9
The duty of the State is preeminently "to serve . . . the people, and so also, to "promote which the government was called upon to enter optionally and only
a just and dynamic social order . . . through policies that provide adequate social because it was better equipped to administer for the public welfare than in
services. . . . and an improved quality of life for all. 10 any private individual or group of individuals," continue to lose their well-
defined boundaries and to be absorbed within activities that the
The objectives of government, as expressed in the Charter, are, among other things, "a government must undertake in its sovereign capacity if it is to meet the
more equitable distribution of opportunities, income, and wealth . . . [and] a sustained increasing social challenges of the times. Here as almost everywhere else
increase in the amount of goods and services produced by the nation for the benefit of the the tendency is undoubtedly towards a greater socialization of economic
people . . . "11 With respect in particular to property, the Constitution decrees: forces. Here of course this development was envisioned, indeed adopted
as a national policy, by the Constitution itself in its declaration of principle
concerning the promotion of social justice. 15
Sec. 6. The use of property bears a social function, and all economic
agents shall contribute to the common good. Individuals and private groups,
including corporations, cooperatives, and similar collective organizations, The requirements of social justice and the necessity for a redistribution of the national
shall have the right to own, establish, and operate economic enterprises, wealth and economic opportunity find in fact a greater emphasis in the 1987 Constitution,
subject to the duty of the State to promote distributive justice and to notwithstanding the novel concepts inscribed there. 16 And two decades after this Court
intervene when the common good so demands. 12 wrote it, ACCFAs message remains the same and its lesson holds true as ever.

There can hardly be any valid argument against providing for public corresponding, free of The Court is not of the thinking that the act complained of is equivalent to a taking without
charge. It is compatible with State aims to serve the people under the Constitution, and just compensation. Albeit we have held that "[w]here the owner is deprived of the ordinary
certainly, amid these hard times, the State can do no less. and beneficial use of his property or of its value by its being diverted to public use, there
is taking within the constitutional sense, 17 it does not seem to us that the Department of
Transportation and Communication, by providing for free public correspondence, is guilty
of an uncompensated taking. Rather, the Government merely built a bridge that made the
42 | P a g e
NAT RES & ENVI LAW CASES 2A_2023
boat obsolete, although not entirely useless. Certainly, the owner of the boat can not
charge the builder of the bridge for lost income. And certainly, the Government has all the
right to build the bridge.

WHEREFORE, the petition is DISMISSED. No costs.

SO ORDERED.

43 | P a g e

You might also like